NUR 245 Final Exam Testbank Questions

Ace your homework & exams now with Quizwiz!

During the assessment of deep tendon reflexes, the nurse finds that a patients responses are bilaterally normal. What number is used to indicate normal deep tendon reflexes when the documenting this finding? ____+

ANS: 2 Responses to assessment of deep tendon reflexes are graded on a 4-point scale. A rating of 2+ indicates normal or average response. A rating of 0 indicates no response, and a rating of 4+ indicates very brisk, hyperactive response with clonus, which is indicative of disease.

The nurse is reviewing a patients medical record and notes that he is in a coma. Using the Glasgow Coma Scale, which number indicates that the patient is in a coma? a. 6 b. 12 c. 15 d. 24

ANS: A A fully alert, normal person has a score of 15, whereas a score of 7 or less reflects coma on the Glasgow Coma Scale

In the assessment of a 1-month-old infant, the nurse notices a lack of response to noise or stimulation. The mother reports that in the last week he has been sleeping all of the time, and when he is awake all he does is cry. The nurse hears that the infants cries are very high pitched and shrill. What should be the nurses appropriate response to these findings? a. Refer the infant for further testing. b. Talk with the mother about eating habits. c. Do nothing; these are expected findings for an infant this age. d. Tell the mother to bring the baby back in 1 week for a recheck.

ANS: A A high-pitched, shrill cry or cat-sounding screech occurs with central nervous system damage. Lethargy, hyporeactivity, and hyperirritability, as well as the parents report of significant changes in behavior all warrant referral. The other options are not correct responses.

A 21-year-old patient has a head injury resulting from trauma and is unconscious. There are no other injuries. During the assessment what would the nurse expect to find when testing the patients deep tendon reflexes? a. Reflexes will be normal. b. Reflexes cannot be elicited. c. All reflexes will be diminished but present. d. Some reflexes will be present, depending on the area of injury.

ANS: A A reflex is a defense mechanism of the nervous system. It operates below the level of conscious control and permits a quick reaction to potentially painful or damaging situations.

The nurse knows that the best time to assess a womans blood pressure during an initial prenatal visit is: a. At the end of the examination when she will be the most relaxed. b. At the beginning of the interview as a nonthreatening method of gaining rapport. c. During the middle of the physical examination when she is the most comfortable. d. Before beginning the pelvic examination because her blood pressure will be higher after the pelvic examination.

ANS: A Assessing the womans blood pressure at the end of the examination, when it is hoped that she will be most relaxed, is the best time to assess blood pressure. The other options are not correct.

To assess the head control of a 4-month-old infant, the nurse lifts up the infant in a prone position while supporting his chest. The nurse looks for what normal response? The infant: a. Raises the head, and arches the back. b. Extends the arms, and drops down the head. c. Flexes the knees and elbows with the back straight. d. Holds the head at 45 degrees, and keeps the back straight.

ANS: A At 3 months of age, the infant raises the head and arches the back as if in a swan dive. This response is the Landau reflex, which persists until 1 years of age. The other responses are incorrect.

During a speculum inspection of the vagina, the nurse would expect to see what at the end of the vaginal canal? a. Cervix b. Uterus c. Ovaries d. Fallopian tubes

ANS: A At the end of the canal, the uterine cervix projects into the vagina.

The nurse should use which test to check for large amounts of fluid around the patella? a. Ballottement b. Tinel sign c. Phalen test d. McMurray test

ANS: A Ballottement of the patella is reliable when large amounts of fluid are present. The Tinel sign and the Phalen test are used to check for carpal tunnel syndrome. The McMurray test is used to test the knee for a torn meniscus.

After the health history has been obtained and before beginning the physical examination, the nurse should first ask the patient to: a. Empty the bladder. b. Completely disrobe. c. Lie on the examination table. d. Walk around the room.

ANS: A Before beginning the examination, the nurse should ask the person to empty the bladder (save the specimen if needed), disrobe except for underpants, put on a gown, and sit with the legs dangling off side of the bed or table.

During an inspection of a patients face, the nurse notices that the facial features are symmetric. This finding indicates which cranial nerve is intact? a. VII b. IX c. XI d. XII

ANS: A Cranial nerve VII is responsible for facial symmetry.

A patients uvula raises midline when she says ahh, and she has a positive gag reflex. The nurse has just tested which cranial nerves? a. IX and X b. IX and XII c. X and XII d. XI and XII

ANS: A Cranial nerves IX and X are being tested by having the patient say ahh, noting the mobility of the uvula, and when assessing the patients gag reflex.

A patient states, I can hear a crunching or grating sound when I kneel. She also states that it is very difficult to get out of bed in the morning because of stiffness and pain in my joints. The nurse should assess for signs of what problem? a. Crepitation b. Bone spur c. Loose tendon d. Fluid in the knee joint

ANS: A Crepitation is an audible and palpable crunching or grating that accompanies movement and occurs when articular surfaces in the joints are roughened, as with rheumatoid arthritis. The other options are not correct.

The nurse should use which location for eliciting deep tendon reflexes? a. Achilles b. Femoral c. Scapular d. Abdominal

ANS: A Deep tendon reflexes are elicited in the biceps, triceps, brachioradialis, patella, and Achilles heel.

A 59-year-old patient has been diagnosed with prostatitis and is being seen at the clinic for complaints of burning and pain during urination. He is experiencing: a. Dysuria. b. Nocturia. c. Polyuria. d. Hematuria.

ANS: A Dysuria (burning with urination) is common with acute cystitis, prostatitis, and urethritis. Nocturia is voiding during the night. Polyuria is voiding in excessive quantities. Hematuria is voiding with blood in the urine.

The nurse is examining a 6-month-old infant and places the infants feet flat on the table and flexes his knees up. The nurse notes that the right knee is significantly lower than the left. Which of these statements is true of this finding? a. This finding is a positive Allis sign and suggests hip dislocation. b. The infant probably has a dislocated patella on the right knee. c. This finding is a negative Allis sign and normal for an infant of this age. d. The infant should return to the clinic in 2 weeks to see if his condition has changed.

ANS: A Finding one knee significantly lower than the other is a positive Allis sign and suggests hip dislocation. Normally, the tops of the knees are at the same elevation. The other statements are not correct.

A patient tells the nurse that she is having a hard time bringing her hand to her mouth when she eats or tries to brush her teeth. The nurse knows that for her to move her hand to her mouth, she must perform which movement? a. Flexion b. Abduction c. Adduction d. Extension

ANS: A Flexion, or bending a limb at a joint, is required to move the hand to the mouth. Extension is straightening a limb at a joint. Moving a limb toward the midline of the body is called adduction; abduction is moving a limb away from the midline of the body.

A female patient tells the nurse that she has four children and has had three pregnancies. How should the nurse document this? a. Gravida 3, para 4 b. Gravida 4, para 3 c. This information cannot be documented using the terms gravida and para. d. The patient seems to be confused about how many times she has been pregnant.

ANS: A Gravida refers to the number of pregnancies, and para refers to the number of children. One pregnancy was with twins.

During a health history of a patient who complains of chronic constipation, the patient asks the nurse about high-fiber foods. The nurse relates that an example of a high-fiber food would be: a. Broccoli. b. Hamburger. c. Iceberg lettuce. d. Yogurt.

ANS: A High-fiber foods are either soluble type (e.g., beans, prunes, barley, broccoli) or insoluble type (e.g., cereals, wheat germ). The other examples are not considered high-fiber foods.

In a person with an upper motor neuron lesion such as a cerebrovascular accident, which of these physical assessment findings should the nurse expect? a. Hyperreflexia b. Fasciculations c. Loss of muscle tone and flaccidity d. Atrophy and wasting of the muscles

ANS: A Hyperreflexia, diminished or absent superficial reflexes, and increased muscle tone or spasticity can be expected with upper motor neuron lesions. The other options reflect a lesion of lower motor neurons

The mother of a 5-year-old girl tells the nurse that she has noticed her daughter scratching at her bottom a lot the last few days. During the assessment, the nurse finds redness and raised skin in the anal area. This finding most likely indicates: a. Pinworms. b. Chickenpox. c. Constipation. d. Bacterial infection.

ANS: A In children, pinworms are a common cause of intense itching and irritated anal skin. The other options are not correct.

An older man is concerned about his sexual performance. The nurse knows that in the absence of disease, a withdrawal from sexual activity later in life may be attributable to: a. Side effects of medications. b. Decreased libido with aging. c. Decreased sperm production. d. Decreased pleasure from sexual intercourse.

ANS: A In the absence of disease, a withdrawal from sexual activity may be attributable to side effects of medications such as antihypertensives, antidepressants, sedatives, psychotropics, antispasmotics, tranquilizers or narcotics, and estrogens. The other options are not correct.

When assessing a newborn infants genitalia, the nurse notices that the genitalia are somewhat engorged. The labia majora are swollen, the clitoris looks large, and the hymen is thick. The vaginal opening is difficult to visualize. The infants mother states that she is worried about the labia being swollen. The nurse should reply: a. This is a normal finding in newborns and should resolve within a few weeks. b. This finding could indicate an abnormality and may need to be evaluated by a physician. c. We will need to have estrogen levels evaluated to ensure that they are within normal limits. d. We will need to keep close watch over the next few days to see if the genitalia decrease in size.

ANS: A It is normal for a newborns genitalia to be somewhat engorged. A sanguineous vaginal discharge or leukorrhea is normal during the first few weeks because of the maternal estrogen effect. During the early weeks, the genital engorgement resolves, and the labia minora atrophy and remain small until puberty

The functional units of the musculoskeletal system are the: a. Joints. b. Bones. c. Muscles. d. Tendons.

ANS: A Joints are the functional units of the musculoskeletal system because they permit the mobility needed to perform the activities of daily living. The skeleton (bones) is the framework of the body. The other options are not correct.

During an examination, which tests will the nurse collect to screen for cervical cancer? a. Endocervical specimen, cervical scrape, and vaginal pool b. Endocervical specimen, vaginal pool, and acetic acid wash c. Endocervical specimen, potassium hydroxide (KOH) preparation, and acetic acid wash d. Cervical scrape, acetic acid wash, saline mount (wet prep)

ANS: A Laboratories may vary in method, but usually the test consists of three specimens: endocervical specimen, cervical scrape, and vaginal pool. The other tests (acetic acid wash, KOH preparation, and saline mount) are used to test for sexually transmitted infections.

A woman who is 8 months pregnant comments that she has noticed a change in her posture and is having lower back pain. The nurse tells her that during pregnancy, women have a posture shift to compensate for the enlarging fetus. This shift in posture is known as: a. Lordosis. b. Scoliosis. c. Ankylosis. d. Kyphosis.

ANS: A Lordosis compensates for the enlarging fetus, which would shift the center of balance forward. This shift in balance, in turn, creates a strain on the low back muscles, felt as low back pain during late pregnancy by some women. Scoliosis is lateral curvature of portions of the spine; ankylosis is extreme flexion of the wrist, as observed with severe rheumatoid arthritis; and kyphosis is an enhanced thoracic curvature of the spine.

A woman is in the clinic for an annual gynecologic examination. The nurse should plan to begin the interview with the: a. Menstrual history, because it is generally nonthreatening. b. Obstetric history, because it includes the most important information. c. Urinary system history, because problems may develop in this area as well. d. Sexual history, because discussing it first will build rapport.

ANS: A Menstrual history is usually nonthreatening and therefore a good topic with which to begin the interview. Obstetric, urinary, and sexual histories are also part of the interview but not necessarily the best topics with which to start.

To test for gross motor skill and coordination of a 6-year-old child, which of these techniques would be appropriate? Ask the child to: a. Hop on one foot. b. Stand on his head. c. Touch his finger to his nose. d. Make funny faces at the nurse.

ANS: A Normally, a child can hop on one foot and can balance on one foot for approximately 5 seconds by 4 years of age and can balance on one foot for 8 to 10 seconds at 5 years of age. Children enjoy performing these tests. Failure to hop after 5 years of age indicates incoordination of gross motor skills. Asking the child to touch his or her finger to the nose checks fine motor coordination; and asking the child to make funny faces tests CN VII. Asking a child to stand on his or her head is not appropriate.

When performing a genitourinary assessment, the nurse notices that the urethral meatus is ventrally positioned. This finding is: a. Called hypospadias. b. A result of phimosis. c. Probably due to a stricture. d. Often associated with aging.

ANS: A Normally, the urethral meatus is positioned just about centrally. Hypospadias is the ventral location of the urethral meatus. The position of the meatus does not change with aging. Phimosis is the inability to retract the foreskin. A stricture is a narrow opening of the meatus.

What should the nurse assess before entering the patients room on morning rounds? a. Posted conditions, such as isolation precautions b. Patients input and output chart from the previous shift c. Patients general appearance d. Presence of any visitors in the room

ANS: A On the way to the patients room, the nurse should assess the presence of conditions such as isolation precautions, latex allergies, or fall precautions.

During an assessment of a hospitalized patient, the nurse pinches a fold of skin under the clavicle or on the forearm to test the: a. Mobility and turgor. b. Patients response to pain. c. Percentage of the patients fat-to-muscle ratio. d. Presence of edema.

ANS: A Pinching a fold of skin under the clavicle or on the forearm is done by the nurse to determine mobility and turgor.

The nurse is preparing to interview a postmenopausal woman. Which of these statements is true as it applies to obtaining the health history of a postmenopausal woman? a. The nurse should ask a postmenopausal woman if she has ever had vaginal bleeding. b. Once a woman reaches menopause, the nurse does not need to ask any history questions. c. The nurse should screen for monthly breast tenderness. d. Postmenopausal women are not at risk for contracting STIs; therefore, these questions can be omitted.

ANS: A Postmenopausal bleeding warrants further workup and referral. The other statements are not true.

While obtaining a health history of a 3-month-old infant from the mother, the nurse asks about the infants ability to suck and grasp the mothers finger. What is the nurse assessing? a. Reflexes b. Intelligence c. CNs d. Cerebral cortex function

ANS: A Questions regarding reflexes include such questions as, What have you noticed about the infants behavior, Are the infants sucking and swallowing seem coordinated, and Does the infant grasp your finger? The other responses are incorrect.

The nurse is inspecting the scrotum and testes of a 43-year-old man. Which finding would require additional follow-up and evaluation? a. Skin on the scrotum is taut. b. Left testicle hangs lower than the right testicle. c. Scrotal skin has yellowish 1-cm nodules that are firm and nontender. d. Testes move closer to the body in response to cold temperatures.

ANS: A Scrotal swelling may cause the skin to be taut and to display pitting edema. Normal scrotal skin is rugae, and asymmetry is normal with the left scrotal half usually lower than the right. The testes may move closer to the body in response to cold temperatures.

During the assessment of an 80-year-old patient, the nurse notices that his hands show tremors when he reaches for something and his head is always nodding. No associated rigidity is observed with movement. Which of these statements is most accurate? a. These findings are normal, resulting from aging. b. These findings could be related to hyperthyroidism. c. These findings are the result of Parkinson disease. d. This patient should be evaluated for a cerebellar lesion.

ANS: A Senile tremors occasionally occur. These benign tremors include an intention tremor of the hands, head nodding (as if saying yes or no), and tongue protrusion. Tremors associated with Parkinson disease include rigidity, slowness, and a weakness of voluntary movement. The other responses are incorrect.

When performing an external genitalia examination of a 10-year-old girl, the nurse notices that no pubic hair has grown in and the mons and the labia are covered with fine vellus hair. These findings are consistent with stage _____ of sexual maturity, according to the Sexual Maturity Rating scale. a. 1 b. 2 c. 3 d. 4

ANS: A Sexual Maturity Rating stage 1 is the preadolescent stage. There is no pubic hair, and the mons and labia are covered with fine, vellus hair as on the abdomen

The nurse is testing the deep tendon reflexes of a 30-year-old woman who is in the clinic for an annual physical examination. When striking the Achilles heel and quadriceps muscle, the nurse is unable to elicit a reflex. The nurses next response should be to: a. Ask the patient to lock her fingers and pull. b. Complete the examination, and then test these reflexes again. c. Refer the patient to a specialist for further testing. d. Document these reflexes as 0 on a scale of 0 to 4+.

ANS: A Sometimes the reflex response fails to appear. Documenting the reflexes as absent is inappropriate this soon in the examination. The nurse should try to further encourage relaxation, varying the persons position or increasing the strength of the blow. Reinforcement is another technique to relax the muscles and enhance the response. The person should be asked to perform an isometric exercise in a muscle group somewhat away from the one being tested. For example, to enhance a patellar reflex, the person should be asked to lock the fingers together and pull.

A male patient with possible fertility problems asks the nurse where sperm is produced. The nurse knows that sperm production occurs in the: a. Testes. b. Prostate. c. Epididymis. d. Vas deferens.

ANS: A Sperm production occurs in the testes, not in the other structures listed.

A 70-year-old man is visiting the clinic for difficulty in passing urine. In the health history, he indicates that he has to urinate frequently, especially at night. He has burning when he urinates and has noticed pain in his back. Considering this history, what might the nurse expect to find during the physical assessment? a. Asymmetric, hard, and fixed prostate gland b. Occult blood and perianal pain to palpation c. Symmetrically enlarged, soft prostate gland d. Soft nodule protruding from the rectal mucosa

ANS: A Subjective symptoms of carcinoma of the prostate include frequency, nocturia, hematuria, weak stream, hesitancy, pain or burning on urination, and continuous pain in lower back, pelvis, and thighs. Objective symptoms of carcinoma of the prostate include a malignant neoplasm that often starts as a single hard nodule on the posterior surface, producing asymmetry and a change in consistency. As it invades normal tissue, multiple hard nodules appear, or the entire gland feels stone hard and fixed.

The nurse is performing a vision examination. Which of these charts is most widely used for vision examinations? a. Snellen b. Shetllen c. Smoollen d. Schwellon

ANS: A The Snellen eye chart is most widely used for vision examinations. The other options are not tests for vision examinations.

An 11-year-old girl is in the clinic for a sports physical examination. The nurse notices that she has begun to develop breasts, and during the conversation the girl reveals that she is worried about her development. The nurse should use which of these techniques to best assist the young girl in understanding the expected sequence for development? The nurse should: a. Use the Tanner scale on the five stages of sexual development. b. Describe her development and compare it with that of other girls her age. c. Use the Jacobsen table on expected development on the basis of height and weight data. d. Reassure her that her development is within normal limits and tell her not to worry about the next step.

ANS: A The Tanner scale on the five stages of pubic hair development is helpful in teaching girls the expected sequence of sexual development. The other responses are not appropriate.

The ankle joint is the articulation of the tibia, fibula, and: a. Talus. b. Cuboid. c. Calcaneus. d. Cuneiform bones.

ANS: A The ankle or tibiotalar joint is the articulation of the tibia, fibula, and talus. The other bones listed are foot bones and not part of the ankle joint.

The nurse knows that determining whether a person is oriented to his or her surroundings will test the functioning of which structure(s)? a. Cerebrum b. Cerebellum c. CNs d. Medulla oblongata

ANS: A The cerebral cortex is responsible for thought, memory, reasoning, sensation, and voluntary movement. The other structures are not responsible for a persons level of consciousness.

A 62-year-old man is experiencing fever, chills, malaise, urinary frequency, and urgency. He also reports urethral discharge and a dull aching pain in the perineal and rectal area. These symptoms are most consistent with which condition? a. Prostatitis b. Polyps c. Carcinoma of the prostate d. BPH

ANS: A The common presenting symptoms of prostatitis are fever, chills, malaise, and urinary frequency and urgency. The individual may also have dysuria, urethral discharge, and a dull aching pain in the perineal and rectal area. These symptoms are not consistent with polyps.

The nurse is caring for a newborn infant. Thirty hours after birth, the infant passes a dark green meconium stool. The nurse recognizes this is important because the: a. Stool indicates anal patency. b. Dark green color indicates occult blood in the stool. c. Meconium stool can be reflective of distress in the newborn. d. Newborn should have passed the first stool within 12 hours after birth.

ANS: A The first stool passed by the newborn is dark green meconium and occurs within 24 to 48 hours of birth, indicating anal patency. The other responses are not correct.

The wife of a 65-year-old man tells the nurse that she is concerned because she has noticed a change in her husbands personality and ability to understand. He also cries very easily and becomes angry. The nurse recalls that the cerebral lobe responsible for these behaviors is the __________ lobe. a. Frontal b. Parietal c. Occipital d. Temporal

ANS: A The frontal lobe has areas responsible for personality, behavior, emotions, and intellectual function. The parietal lobe has areas responsible for sensation; the occipital lobe is responsible for visual reception; and the temporal lobe is responsible for hearing, taste, and smell.

The nurse is checking the range of motion in a patients knee and knows that the knee is capable of which movement(s)? a. Flexion and extension b. Supination and pronation c. Circumduction d. Inversion and eversion

ANS: A The knee is a hinge joint, permitting flexion and extension of the lower leg on a single plane. The knee is not capable of the other movements listed.

During an examination, the nurse observes a female patients vestibule and expects to see the: a. Urethral meatus and vaginal orifice. b. Vaginal orifice and vestibular (Bartholin) glands. c. Urethral meatus and paraurethral (Skene) glands. d. Paraurethral (Skene) and vestibular (Bartholin) glands.

ANS: A The labial structures encircle a boat-shaped space, or cleft, termed the vestibule. Within the vestibule are numerous openings. The urethral meatus and vaginal orifice are visible. The ducts of the paraurethral (Skene) glands and the vestibular (Bartholin) glands are present but not visible.

When performing a musculoskeletal assessment, the nurse knows that the correct approach for the examination should be: a. Proximal to distal. b. Distal to proximal. c. Posterior to anterior. d. Anterior to posterior.

ANS: A The musculoskeletal assessment should be performed in an orderly approach, head to toe, proximal to distal, from the midline outward. The other options are not correct.

During auscultation of fetal heart tones (FHTs), the nurse determines that the heart rate is 136 beats per minute. The nurses next action should be to: a. Document the results, which are within normal range. b. Take the maternal pulse to verify these findings as the uterine souffle. c. Have the patient change positions and count the FHTs again. d. Immediately notify the physician for possible fetal distress.

ANS: A The normal fetal heart rate is between 110 and 160 beats per minute. The nurse should document the results as within the normal range. The other options are not correct.

The nurse is preparing to palpate the rectum and should use which of these techniques? The nurse should: a. Flex the finger, and slowly insert it toward the umbilicus. b. First instruct the patient that this procedure will be painful. c. Insert an extended index finger at a right angle to the anus. d. Place the finger directly into the anus to overcome the tight sphincter.

ANS: A The nurse should gently place the pad of the index finger against the anal verge. The nurse will feel the sphincter tighten and then relax. As it relaxes, the nurse should flex the tip of the finger and slowly insert it into the anal canal in a direction toward the umbilicus. The nurse should never approach the anus at right angles with the index finger extended; doing so would cause pain. The nurse should instruct the patient that palpation is not painful but may feel like needing to move the bowels.

Which of these statements is most appropriate when the nurse is obtaining a genitourinary history from an older man? a. Do you need to get up at night to urinate? b. Do you experience nocturnal emissions, or wet dreams? c. Do you know how to perform a testicular self-examination? d. Has anyone ever touched your genitals when you did not want them to?

ANS: A The older male patient should be asked about the presence of nocturia. Awaking at night to urinate may be attributable to a diuretic medication, fluid retention from mild heart failure or varicose veins, or fluid ingestion 3 hours before bedtime, especially coffee and alcohol. The other questions are more appropriate for younger men.

When performing an examination of a woman who is 34 weeks pregnant, the nurse notices a midline linear protrusion in the abdomen over the area of the rectus abdominis muscles as the woman raises her head and shoulders off of the bed. Which response by the nurse is correct? a. The presence of diastasis recti should be documented. b. This condition should be discussed with the physician because it will most likely need to be surgically repaired. c. The possibility that the woman has a hernia attributable to the increased pressure within the abdomen from the pregnancy should be suspected. d. The woman should be told that she may have a difficult time with delivery because of the weakness in her abdominal muscles.

ANS: A The separation of the abdominal muscles is called diastasis recti and frequently occurs during pregnancy. The rectus abdominis muscles will return together after pregnancy with abdominal exercise. This condition is not a true hernia.

During an assessment of a 62-year-old man, the nurse notices the patient has a stooped posture, shuffling walk with short steps, flat facial expression, and pill-rolling finger movements. These findings would be consistent with: a. Parkinsonism. b. Cerebral palsy. c. Cerebellar ataxia. d. Muscular dystrophy.

ANS: A The stooped posture, shuffling walk, short steps, flat facial expression, and pill-rolling finger movements are all found in parkinsonism.

Which finding is considered normal and expected when the nurse is performing a physical examination on a pregnant woman? a. Palpable, full thyroid b. Edema in one lower leg c. Significant diffuse enlargement of the thyroid d. Pale mucous membranes of the mouth

ANS: A The thyroid may be palpable during pregnancy. It should feel full, but smooth. Significant diffuse enlargement occurs with hyperthyroidism, thyroiditis, and hypothyroidism. Pale mucous membranes may indicate anemia. Bilateral lower extremity edema is common in pregnancy, but edema with pain in only one leg occurs with deep vein thrombosis.

The uterus is usually positioned tilting forward and superior to the bladder. This position is known as: a. Anteverted and anteflexed. b. Retroverted and anteflexed. c. Retroverted and retroflexed. d. Superiorverted and anteflexed.

ANS: A The uterus is freely movable, not fixed, and usually tilts forward and superior to the bladder (a position labeled as anteverted and anteflexed).

The nurse is reviewing the changes that occur with menopause. Which changes are associated with menopause? a. Uterine and ovarian atrophy, along with a thinning of the vaginal epithelium b. Ovarian atrophy, increased vaginal secretions, and increasing clitoral size c. Cervical hypertrophy, ovarian atrophy, and increased acidity of vaginal secretions d. Vaginal mucosa fragility, increased acidity of vaginal secretions, and uterine hypertrophy

ANS: A The uterus shrinks because of its decreased myometrium. The ovaries atrophy to 1 to 2 cm and are not palpable after menopause. The sacral ligaments relax, and the pelvic musculature weakens; consequently, the uterus droops. The cervix shrinks and looks paler with a thick glistening epithelium. The vaginal epithelium atrophies, becoming thinner, drier, and itchy. The vaginal pH becomes more alkaline, and secretions are decreased, which results in a fragile mucosal surface that is at risk for vaginitis.

During a vaginal examination of a 38-year-old woman, the nurse notices that the vulva and vagina are erythematous and edematous with thick, white, curd-like discharge adhering to the vaginal walls. The woman reports intense pruritus and thick white discharge from her vagina. The nurse knows that these history and physical examination findings are most consistent with which condition? a. Candidiasis b. Trichomoniasis c. Atrophic vaginitis d. Bacterial vaginosis

ANS: A The woman with candidiasis often reports intense pruritus and thick white discharge. The vulva and vagina are erythematous and edematous. The discharge is usually thick, white, and curdlike. Infection with trichomoniasis causes a profuse, watery, gray-green, and frothy discharge. Bacterial vaginosis causes a profuse discharge that has a foul, fishy, rotten odor. Atrophic vaginitis may have a mucoid discharge.

Of the 33 vertebrae in the spinal column, there are: a. 5 lumbar. b. 5 thoracic. c. 7 sacral. d. 12 cervical.

ANS: A There are 7 cervical, 12 thoracic, 5 lumbar, 5 sacral, and 3 to 4 coccygeal vertebrae in the spinal column.

Which of these tests would the nurse use to check the motor coordination of an 11-month-old infant? a. Denver II b. Stereognosis c. Deep tendon reflexes d. Rapid alternating movements

ANS: A To screen gross and fine motor coordination, the nurse should use the Denver II with its age-specific developmental milestones. Stereognosis tests a persons ability to recognize objects by feeling them and is not appropriate for an 11-month-old infant. Testing the deep tendon reflexes is not appropriate for checking motor coordination. Testing rapid alternating movements is appropriate for testing coordination in adults.

During the taking of the health history, a patient tells the nurse that it feels like the room is spinning around me. The nurse would document this finding as: a. Vertigo. b. Syncope. c. Dizziness. d. Seizure activity.

ANS: A True vertigo is rotational spinning caused by a neurologic dysfunction or a problem in the vestibular apparatus or the vestibular nuclei in the brainstem. Syncope is a sudden loss of strength or a temporary loss of consciousness. Dizziness is a lightheaded, swimming sensation. Seizure activity is characterized by altered or loss of consciousness, involuntary muscle movements, and sensory disturbances.

Which of these actions is most appropriate to perform on a 9-month-old infant at a well-child checkup? a. Testing for Ortolani sign b. Assessment for stereognosis c. Blood pressure measurement d. Assessment for the presence of the startle reflex

ANS: A Until the age of 12 months, the infant should be assessed for Ortolani sign. If Ortolani sign is present, then it could indicate the presence of a dislocated hip. The other tests are not appropriate for a 9-month-old child.

During an examination, the patient tells the nurse that she sometimes feels as if objects are spinning around her. The nurse would document that she occasionally experiences: a. Vertigo. b. Tinnitus. c. Syncope. d. Dizziness.

ANS: A Vertigo is the sensation of a person moving around in space (subjective) or of the person sensing objects moving around him or her (objective) and is a result of a disturbance of equilibratory apparatus

A 59-year-old patient has a herniated intervertebral disk. Which of the following findings should the nurse expect to see on physical assessment of this individual? a. Hyporeflexia b. Increased muscle tone c. Positive Babinski sign d. Presence of pathologic reflexes

ANS: A With a herniated intervertebral disk or lower motor neuron lesion, loss of tone, flaccidity, atrophy, fasciculations, and hyporeflexia or areflexia are demonstrated. No Babinski sign or pathologic reflexes would be observed (see Table 23-7). The other options reflect a lesion of upper motor neurons.

During a physical examination, the nurse finds that a male patients foreskin is fixed and tight and will not retract over the glans. The nurse recognizes that this condition is: a. Phimosis. b. Epispadias. c. Urethral stricture. d. Peyronie disease

ANS: A With phimosis, the foreskin is nonretractable, forming a pointy tip of the penis with a tiny orifice at the end of the glans. The foreskin is advanced and so tight that it is impossible to retract over the glans. This condition may be congenital or acquired from adhesions related to infection.

The nurse is assessing the IV infusion at the beginning of the shift. Which factors should be included in the assessment of the infusion? Select all that apply. a. Proper IV solution is infusing, according to the physicians orders. b. The IV solution is infusing at the proper rate, according to physicians orders. c. The infusion is proper, according to the nurses assessment of the patients needs. d. Capillary refill in the fingers is checked and noted. e. The IV site date is noted. f. Whether the patient is sufficiently voiding is noted.

ANS: A, B, C, E The nurse should verify that the proper IV solution is hanging and is flowing at the proper rate according to the physicians orders and the nurses own assessment of the patients needs. In addition, the nurse should note the date of the IV site and surrounding skin condition. Checking capillary refill is part of the cardiovascular assessment; checking the patients voiding is part of the genitourinary assessment.

The nurse is performing a digital examination of a patients prostate gland and notices that a normal prostate gland includes which of the following characteristics? Select all that apply. a. 1 cm protrusion into the rectum b. Heart-shaped with a palpable central groove c. Flat shape with no palpable groove d. Boggy with a soft consistency e. Smooth surface, elastic, and rubbery consistency f. Fixed mobility

ANS: A, B, E The size of a normal prostate gland should be 2.5 cm long by 4 cm wide and should not protrude more than 1 cm into the rectum. The prostate should be heart-shaped, with a palpable central groove, a smooth surface, and elastic with a rubbery consistency. Abnormal findings include a flat shape with no palpable groove, boggy with a soft consistency, and fixed mobility.

A 16-year-old boy is brought to the clinic for a problem that he refused to let his mother see. The nurse examines him, and finds that he has scrotal swelling on the left side. He had the mumps the previous week, and the nurse suspects that he has orchitis. Which of the following assessment findings support this diagnosis? Select all that apply. a. Swollen testis b. Mass that transilluminates c. Mass that does not transilluminate d. Scrotum that is nontender upon palpation e. Scrotum that is tender upon palpation f. Scrotal skin that is reddened

ANS: A, C, E, F With orchitis, the testis is swollen, with a feeling of weight, and is tender or painful. The mass does not transilluminate, and the scrotal skin is reddened. Transillumination of a mass occurs with a hydrocele, not orchitis.

A 55-year-old man is in the clinic for a yearly checkup. He is worried because his father died of prostate cancer. The nurse knows which tests should be performed at this time? Select all that apply. a. Blood test for prostate-specific antigen (PSA) b. Urinalysis c. Transrectal ultrasound d. Digital rectal examination (DRE) e. Prostate biopsy

ANS: A, D Prostate cancer is typically detected by testing the blood for PSA or by a DRE. It is recommended that both PSA and DRE be offered to men annually, beginning at age 50 years. If the PSA is elevated, then further laboratory work or a transrectal ultrasound (TRUS) and biopsy may be recommended.

The nurse is completing an assessment on a patient who was just admitted from the emergency department. Which assessment findings would require immediate attention? Select all that apply. a. Temperature: 38.6 C b. Systolic blood pressure: 150 mm Hg c. Respiratory rate: 22 breaths per minute d. Heart rate: 130 beats per minute e. Oxygen saturation: 95% f. Sudden restlessness

ANS: A, D, F The following examination findings require immediate attention: - High or low temperature: (36.1 C or 37.8 C) - High or low blood pressure: (systolic pressure 90 mm Hg or 160 mm Hg) - High or low number of respirations: (12 or 28 breaths per minute) - High or low heart rate: (60 or 90 beats per minute) - Oxygen saturation: 92% - Sudden restlessness or anxiety, altered level of consciousness, confusion, or difficulty in arousing

During a digital examination of the rectum, the nurse notices that the patient has hard feces in the rectum. The patient complains of feeling full, has a distended abdomen, and states that she has not had a bowel movement for several days. The nurse suspects which condition? a. Rectal polyp b. Fecal impaction c. Rectal abscess d. Rectal prolapse

ANS: B A fecal impaction is a collection of hard, desiccated feces in the rectum. The obstruction often results from decreased bowel motility, in which more water is reabsorbed from the stool.

A 40-year-old black man is in the office for his annual physical examination. Which statement regarding the PSA blood test is true, according to the American Cancer Society? The PSA: a. Should be performed with this visit. b. Should be performed at age 45 years. c. Should be performed at age 50 years. d. Is only necessary if a family history of prostate cancer exists.

ANS: B According to the American Cancer Society (2006), the PSA blood test should be performed annually for black men beginning at age 45 years and annually for all other men over age 50 years.

During a discussion for a mens health group, the nurse relates that the group with the highest incidence of prostate cancer is: a. Asian Americans. b. Blacks. c. American Indians. d. Hispanics.

ANS: B According to the American Cancer Society (2010), black men have a higher rate of prostate cancer than other racial groups.

When performing a genital examination on a 25-year-old man, the nurse notices deeply pigmented, wrinkled scrotal skin with large sebaceous follicles. On the basis of this information, the nurse would: a. Squeeze the glans to check for the presence of discharge. b. Consider this finding as normal, and proceed with the examination. c. Assess the testicles for the presence of masses or painless lumps. d. Obtain a more detailed history, focusing on any scrotal abnormalities the patient has noticed.

ANS: B After adolescence, the scrotal skin is deeply pigmented and has large sebaceous follicles and appears corrugated.

A patient has been diagnosed with osteoporosis and asks the nurse, What is osteoporosis? The nurse explains that osteoporosis is defined as: a. Increased bone matrix. b. Loss of bone density. c. New, weaker bone growth. d. Increased phagocytic activity.

ANS: B After age 40 years, a loss of bone matrix (resorption) occurs more rapidly than new bone formation. The net effect is a gradual loss of bone density, or osteoporosis. The other options are not correct.

An imaginary line connecting the highest point on each iliac crest would cross the __________ vertebra. a. First sacral b. Fourth lumbar c. Seventh cervical d. Twelfth thoracic

ANS: B An imaginary line connecting the highest point on each iliac crest crosses the fourth lumbar vertebra. The other options are not correct.

During examination, the nurse finds that a patient is unable to distinguish objects placed in his hand. The nurse would document: a. Stereognosis. b. Astereognosis. c. Graphesthesia. d. Agraphesthesia.

ANS: B Astereognosis is the inability to identify correctly an object placed in the hand

A 5-year-old child is in the clinic for a checkup. The nurse would expect him to: a. Need to be held on his mothers lap. b. Be able to sit on the examination table. c. Be able to stand on the floor for the examination. d. Be able to remain alone in the examination room.

ANS: B At 4 or 5 years old, a child usually feels comfortable on the examination table. Older infants and young children aged 6 months to 2 or 3 years should be positioned in the parents lap.

The assessment of a 60-year-old patient has taken longer than anticipated. In testing his pain perception, the nurse decides to complete the test as quickly as possible. When the nurse applies the sharp point of the pin on his arm several times, he is only able to identify these as one very sharp prick. What would be the most accurate explanation for this? a. The patient has hyperesthesia as a result of the aging process. b. This response is most likely the result of the summation effect. c. The nurse was probably not poking hard enough with the pin in the other areas. d. The patient most likely has analgesia in some areas of arm and hyperalgesia in others.

ANS: B At least 2 seconds should be allowed to elapse between each stimulus to avoid summation. With summation, frequent consecutive stimuli are perceived as one strong stimulus. The other responses are incorrect.

A 25-year-old woman is in the clinic for her first prenatal visit. The nurse will prepare to obtain which laboratory screening test at this time? a. Urine toxicology b. Complete blood cell count c. Alpha-fetoprotein d. Carrier screening for cystic fibrosis

ANS: B At the onset of pregnancy, a routine prenatal panel usually includes a complete blood cell count, serologic testing, rubella antibodies, hepatitis B screening, blood type and Rhesus factor, and antibody screen. A clean-catch urine sample is collected for urinalysis to rule out cystitis. Urine toxicology, although beneficial for women if active substance abuse is suspected or known, is not routinely performed. In the second trimester, maternal serum is analyzed for alpha-fetoprotein. Carrier screening for cystic fibrosis is offered to check whether a person carries the abnormal gene that causes cystic fibrosis but is not part of routine testing.

When auscultating the anterior thorax of a pregnant woman, the nurse notices the presence of a murmur over the second, third, and fourth intercostal spaces. The murmur is continuous but can be obliterated by pressure with the stethoscope or finger on the thorax just lateral to the murmur. The nurse interprets this finding to be: a. Murmur of aortic stenosis. b. Most likely a mammary souffle. c. Associated with aortic insufficiency. d. Indication of a patent ductus arteriosus.

ANS: B Blood flow through the blood vessels, specifically the internal mammary artery, can often be heard over the second, third, and fourth intercostal spaces. This finding is called a mammary souffle, but it may be mistaken for a cardiac murmur. The other options are incorrect.

When examining the face of a woman who is 28 weeks pregnant, the nurse notices the presence of a butterfly-shaped increase in pigmentation on the face. The proper term for this finding in the documentation is: a. Striae. b. Chloasma. c. Linea nigra. d. Mask of pregnancy.

ANS: B Chloasma is a butterfly-shaped increase in pigmentation on the face. It is known as the mask of pregnancy, but when documenting, the nurse should use the correct medical term, chloasma. Striae is the term for stretch marks. The linea nigra is a hyperpigmented line that begins at the sternal notch and extends down the abdomen through the umbilicus to the pubis.

A woman at 25 weeks gestation comes to the clinic for her prenatal visit. The nurse notices that her face and lower extremities are swollen, and her blood pressure is 154/94 mm Hg. The woman states that she has had headaches and blurry vision but thought she was just tired. What should the nurse suspect? a. Eclampsia b. Preeclampsia c. Diabetes type 1 d. Preterm labor

ANS: B Classic symptoms of preeclampsia include elevated blood pressure (greater than 140 mm Hg systolic or 90 mm Hg diastolic in a woman with previously normal blood pressure) and proteinuria. Onset and worsening symptoms may be sudden, and subjective signs include headaches and visual changes. Eclampsia is manifested by generalized tonic-clonic seizures. These symptoms are not indicative of diabetes mellitus (type 1 or 2) or preterm labor.

A 40-year-old man has come into the clinic with complaints of extreme pain in his toes. The nurse notices that his toes are slightly swollen, reddened, and warm to the touch. His complaints would suggest: a. Osteoporosis. b. Acute gout. c. Ankylosing spondylitis. d. Degenerative joint disease.

ANS: B Clinical findings for acute gout consist of redness, swelling, heat, and extreme pain like a continuous throbbing. Gout is a metabolic disorder of disturbed purine metabolism, associated with elevated serum uric acid.

The ability that humans have to perform very skilled movements such as writing is controlled by the: a. Basal ganglia. b. Corticospinal tract. c. Spinothalamic tract. d. Extrapyramidal tract.

ANS: B Corticospinal fibers mediate voluntary movement, particularly very skilled, discrete, and purposeful movements, such as writing. The corticospinal tract, also known as the pyramidal tract, is a newer, higher motor system that humans have that permits very skilled and purposeful movements. The other responses are not related to skilled movements.

A 54-year-old woman who has just completed menopause is in the clinic today for a yearly physical examination. Which of these statements should the nurse include in patient education? A postmenopausal woman: a. Is not at any greater risk for heart disease than a younger woman. b. Should be aware that she is at increased risk for dyspareunia because of decreased vaginal secretions. c. Has only stopped menstruating; there really are no other significant changes with which she should be concerned. d. Is likely to have difficulty with sexual pleasure as a result of drastic changes in the female sexual response cycle.

ANS: B Decreased vaginal secretions leave the vagina dry and at risk for irritation and pain with intercourse (dyspareunia). The other statements are incorrect.

The nurse is assessing the neurologic status of a patient who has a late-stage brain tumor. With the reflex hammer, the nurse draws a light stroke up the lateral side of the sole of the foot and inward, across the ball of the foot. In response, the patients toes fan out, and the big toe shows dorsiflexion. The nurse interprets this result as: a. Negative Babinski sign, which is normal for adults. b. Positive Babinski sign, which is abnormal for adults. c. Clonus, which is a hyperactive response. d. Achilles reflex, which is an expected response.

ANS: B Dorsiflexion of the big toe and fanning of all toes is a positive Babinski sign, also called up-going toes. This response occurs with upper motor neuron disease of the corticospinal (or pyramidal) tract and is an abnormal finding for adults.

During inspection of the posterior chest, the nurse should assess for: a. Symmetric expansion. b. Symmetry of shoulders and muscles. c. Tactile fremitus. d. Diaphragmatic excursion.

ANS: B During an inspection of the posterior chest, the nurse should inspect for symmetry of shoulders and muscles, configuration of the thoracic cage, and skin characteristics. Symmetric expansion and tactile fremitus are assessed with palpation; diaphragmatic excursion is assessed with percussion.

During a complete health assessment, how would the nurse test the patients hearing? a. Observing how the patient participates in normal conversation b. Using the whispered voice test c. Using the Weber and Rinne tests d. Testing with an audiometer

ANS: B During the complete health assessment, the nurse should test hearing with the whispered voice test. The other options are not correct.

After completing an assessment of a 60-year-old man with a family history of colon cancer, the nurse discusses with him early detection measures for colon cancer. The nurse should mention the need for a(n): a. Annual proctoscopy. b. Colonoscopy every 10 years. c. Fecal test for blood every 6 months. d. DREs every 2 years.

ANS: B Early detection measures for colon cancer include a DRE performed annually after age 50 years, an annual fecal occult blood test after age 50 years, a sigmoidoscopic examination every 5 years or a colonoscopy every 10 years after age 50 years, and a PSA blood test annually for men over 50 years old, except beginning at age 45 years for black men (American Cancer Society, 2006).

During an examination, the nurse notices severe nystagmus in both eyes of a patient. Which conclusion by the nurse is correct? Severe nystagmus in both eyes: a. Is a normal occurrence. b. May indicate disease of the cerebellum or brainstem. c. Is a sign that the patient is nervous about the examination. d. Indicates a visual problem, and a referral to an ophthalmologist is indicated.

ANS: B End-point nystagmus at an extreme lateral gaze normally occurs; however, the nurse should carefully assess any other nystagmuses. Severe nystagmus occurs with disease of the vestibular system, cerebellum, or brainstem.

A 55-year-old man is experiencing severe pain of sudden onset in the scrotal area. It is somewhat relieved by elevation. On examination the nurse notices an enlarged, red scrotum that is very tender to palpation. Distinguishing the epididymis from the testis is difficult, and the scrotal skin is thick and edematous. This description is consistent with which of these? a. Varicocele b. Epididymitis c. Spermatocele d. Testicular torsion

ANS: B Epididymitis presents as severe pain of sudden onset in the scrotum that is somewhat relieved by elevation. On examination, the scrotum is enlarged, reddened, and exquisitely tender. The epididymis is enlarged and indurated and may be hard to distinguish from the testis. The overlying scrotal skin may be thick and edematous.

A patient is being seen at the clinic for her 10-week prenatal visit. She asks when she will be able to hear the baby's heartbeat. The nurse should reply: a. The baby's heartbeat is not usually heard until the second trimester. b. The baby's heartbeat may be heard anywhere from the ninth to the twelfth week. c. It is often difficult to hear the heartbeat at this point, but we can try. d. It is normal to hear the heartbeat at 6 weeks. We may be able to hear it today.

ANS: B Fetal heart tones can be heard with the use of the Doppler device between 9 and 12 weeks. The other responses are incorrect.

A woman who has had rheumatoid arthritis for years is starting to notice that her fingers are drifting to the side. The nurse knows that this condition is commonly referred to as: a. Radial drift. b. Ulnar deviation. c. Swan-neck deformity. d. Dupuytren contracture.

ANS: B Fingers drift to the ulnar side because of stretching of the articular capsule and muscle imbalance caused by chronic rheumatoid arthritis. A radial drift is not observed.

A patient who is 24 weeks pregnant asks about wearing a seat belt while driving. Which response by the nurse is correct? a. Seat belts should not be worn during pregnancy. b. Place the lap belt below the uterus and use the shoulder strap at the same time. c. Place the lap belt below the uterus but omit the shoulder strap during pregnancy. d. Place the lap belt at your waist above the uterus and use the shoulder strap at the same time.

ANS: B For maternal and fetal safety, the nurse should instruct the woman to place the lap belt below the uterus and to use the shoulder strap. The other instructions are incorrect.

The nurse is preparing to examine the external genitalia of a school-age girl. Which position would be most appropriate in this situation? a. In the parents lap b. In a frog-leg position on the examining table c. In the lithotomy position with the feet in stirrups d. Lying flat on the examining table with legs extended

ANS: B For school-age children, placing them on the examining table in a frog-leg position is best. With toddlers and preschoolers, having the child on the parents lap in a frog-leg position is best.

A patients annual physical examination reveals a lateral curvature of the thoracic and lumbar segments of his spine; however, this curvature disappears with forward bending. The nurse knows that this abnormality of the spine is called: a. Structural scoliosis. b. Functional scoliosis. c. Herniated nucleus pulposus. d. Dislocated hip.

ANS: B Functional scoliosis is flexible and apparent with standing but disappears with forward bending. Structural scoliosis is fixed; the curvature shows both when standing and when bending forward.

A patient tells the nurse that, All my life Ive been called knock knees. The nurse knows that another term for knock knees is: a. Genu varum. b. Genu valgum. c. Pes planus. d. Metatarsus adductus.

ANS: B Genu valgum is also known as knock knees and is present when more than 2.5 cm is between the medial malleoli when the knees are together.

A woman has just been diagnosed with HPV or genital warts. The nurse should counsel her to receive regular examinations because this virus makes her at a higher risk for _______ cancer. a. Uterine b. Cervical c. Ovarian d. Endometrial

ANS: B HPV is the virus responsible for most cases of cervical cancer, not the other options.

A 30-year-old woman is visiting the clinic because of pain in my bottom when I have a bowel movement. The nurse should assess for which problem? a. Pinworms b. Hemorrhoids c. Colon cancer d. Fecal incontinence

ANS: B Having painful bowel movements, known as dyschezia, may be attributable to a local condition (hemorrhoid or fissure) or constipation. The other responses are not correct.

The nurse has administered a pain medication to a patient by an IV infusion. The nurse should reassess the patients response to the pain medication within _____ minutes. a. 5 b. 15 c. 30 d. 60

ANS: B If pain medication is given, then the nurse should reassess the patients response in 15 minutes for IV administration or 1 hour for oral administration.

The nurse is describing how to perform a testicular self-examination to a patient. Which statement is most appropriate? a. A good time to examine your testicles is just before you take a shower. b. If you notice an enlarged testicle or a painless lump, call your health care provider. c. The testicle is egg shaped and movable. It feels firm and has a lumpy consistency. d. Perform a testicular examination at least once a week to detect the early stages of testicular cancer.

ANS: B If the patient notices a firm painless lump, a hard area, or an overall enlarged testicle, then he should call his health care provider for further evaluation. The testicle normally feels rubbery with a smooth surface. A good time to examine the testicles is during the shower or bath, when ones hands are warm and soapy and the scrotum is warm. Testicular self-examination should be performed once a month.

When assessing a patient in the hospital setting, the nurse knows which statement to be true? a. The patient will need a brief assessment at least every 4 hours. b. The patient will need a consistent, specialized examination every 8 hours that focuses on certain parameters. c. The patient will need a complete head-to-toe physical examination every 24 hours. d. Most patients require a minimal examination each shift unless they are in critical condition.

ANS: B In a hospital setting, the patient does not require a complete head-to-toe physical examination during every 24-hour stay. The patient does, however, require a consistent specialized examination every 8 hours that focuses on certain parameters.

During an assessment of a 22-year-old woman who sustained a head injury from an automobile accident 4 hours earlier, the nurse notices the following changes: pupils were equal, but now the right pupil is fully dilated and nonreactive, and the left pupil is 4 mm and reacts to light. What do these findings suggest? a. Injury to the right eye b. Increased intracranial pressure c. Test inaccurately performed d. Normal response after a head injury

ANS: B In a person with a brain injury, a sudden, unilateral, dilated, and nonreactive pupil is ominous. CN III runs parallel to the brainstem. When increasing intracranial pressure pushes down the brainstem (uncal herniation), it puts pressure on CN III, causing pupil dilation. The other responses are incorrect.

The structure that secretes a thin, milky alkaline fluid to enhance the viability of sperm is the: a. Cowper gland. b. Prostate gland. c. Median sulcus. d. Bulbourethral gland.

ANS: B In men, the prostate gland secretes a thin milky alkaline fluid that enhances sperm viability. The Cowper glands (also known as bulbourethral glands) secrete a clear, viscid mucus. The median sulcus is a groove that divides the lobes of the prostate gland and does not secrete fluid.

A nurse is assessing a patients risk of contracting a sexually transmitted infection (STI). An appropriate question to ask would be: a. You know that it's important to use condoms for protection, right? b. Do you use a condom with each episode of sexual intercourse? c. Do you have a sexually transmitted infection? d. You are aware of the dangers of unprotected sex, arent you?

ANS: B In reviewing a patients risk for STIs, the nurse should ask in a nonconfrontational manner whether condoms are being used during each episode of sexual intercourse. Asking a person whether he or she has an infection does not address the risk.

A married couple has come to the clinic seeking advice on pregnancy. They have been trying to conceive for 4 months and have not been successful. What should the nurse do first? a. Ascertain whether either of them has been using broad-spectrum antibiotics. b. Explain that couples are considered infertile after 1 year of unprotected intercourse. c. Immediately refer the woman to an expert in pelvic inflammatory disease the most common cause of infertility. d. Explain that couples are considered infertile after 3 months of engaging in unprotected intercourse and that they will need a referral to a fertility expert.

ANS: B Infertility is considered after 1 year of engaging in unprotected sexual intercourse without conceiving. The other actions are not appropriate.

The nurse knows that testing kinesthesia is a test of a persons: a. Fine touch. b. Position sense. c. Motor coordination. d. Perception of vibration.

ANS: B Kinesthesia, or position sense, is the persons ability to perceive passive movements of the extremities. The other options are incorrect.

An 80-year-old woman is visiting the clinic for a checkup. She states, I cant walk as much as I used to. The nurse is observing for motor dysfunction in her hip and should ask her to: a. Internally rotate her hip while she is sitting. b. Abduct her hip while she is lying on her back. c. Adduct her hip while she is lying on her back. d. Externally rotate her hip while she is standing.

ANS: B Limited abduction of the hip while supine is the most common motion dysfunction found in hip disease. The other options are not correct.

The nurse has just completed an inspection of a nulliparous woman's external genitalia. Which of these would be a description of a finding within normal limits? a. Redness of the labia majora b. Multiple nontender sebaceous cysts c. Discharge that is foul smelling and irritating d. Gaping and slightly shriveled labia majora

ANS: B No lesions should be noted, except for the occasional sebaceous cysts, which are yellowish 1-cm nodules that are firm, nontender, and often multiple. The labia majora are dark pink, moist, and symmetric; redness indicates inflammation or lesions. Discharge that is foul smelling and irritating may indicate infection. In the nulliparous woman, the labia majora meet in the midline, are symmetric and plump.

When performing a scrotal assessment, the nurse notices that the scrotal contents show a red glow with transillumination. On the basis of this finding the nurse would: a. Assess the patient for the presence of a hernia. b. Suspect the presence of serous fluid in the scrotum. c. Consider this finding normal, and proceed with the examination. d. Refer the patient for evaluation of a mass in the scrotum.

ANS: B Normal scrotal contents do not allow light to pass through the scrotum. However, serous fluid does transilluminate and shows as a red glow. Neither a mass nor a hernia would transilluminate.

When the nurse is assessing the deep tendon reflexes (DTRs) on a woman who is 32 weeks pregnant, which of these would be considered a normal finding on a 0 to 4+ scale? a. Absent DTRs b. 2+ c. 4+ d. Brisk reflexes and the presence of clonus

ANS: B Normally during pregnancy, the DTRs are 1+ to 2+ and bilaterally equal. Brisk or greater than 2+ DTRs and the presence of clonus are abnormal and may be associated with an elevated blood pressure and cerebral edema in the preeclamptic woman.

The nurse is palpating a female patients adnexa. The findings include a firm, smooth uterine wall; the ovaries are palpable and feel smooth and firm. The fallopian tube is firm and pulsating. The nurses most appropriate course of action would be to: a. Tell the patient that her examination is normal. b. Give her an immediate referral to a gynecologist. c. Suggest that she return in a month for a recheck to verify the findings. d. Tell the patient that she may have an ovarian cyst that should be evaluated further.

ANS: B Normally, the uterine wall feels firm and smooth, with the contour of the fundus rounded. Ovaries are not often palpable, but when they are, they normally feel smooth, firm, and almond shaped and are highly movable, sliding through the fingers. The fallopian tube is not normally palpable. No other mass or pulsation should be felt. Pulsation or palpable fallopian tube suggests ectopic pregnancy, which warrants immediate referral.

After the examination of an infant, the nurse documents opisthotonos. The nurse recognizes that this finding often occurs with: a. Cerebral palsy. b. Meningeal irritation. c. Lower motor neuron lesion. d. Upper motor neuron lesion.

ANS: B Opisthotonos is a form of spasm in which the head is arched back, and a stiffness of the neck and an extension of the arms and legs are observed. Opisthotonus occurs with meningeal or brainstem irritation

An 85-year-old patient comments during his annual physical examination that he seems to be getting shorter as he ages. The nurse should explain that decreased height occurs with aging because: a. Long bones tend to shorten with age. b. The vertebral column shortens. c. A significant loss of subcutaneous fat occurs. d. A thickening of the intervertebral disks develops.

ANS: B Postural changes are evident with aging; decreased height is most noticeable and is due to shortening of the vertebral column. Long bones do not shorten with age. Intervertebral disks actually get thinner with age. Subcutaneous fat is not lost but is redistributed to the abdomen and hips.

The mother of a 10-year-old boy asks the nurse to discuss the recognition of puberty. The nurse should reply by saying: a. Puberty usually begins around 15 years of age. b. The first sign of puberty is an enlargement of the testes. c. The penis size does not increase until about 16 years of age. d. The development of pubic hair precedes testicular or penis enlargement.

ANS: B Puberty begins sometime between age 9 for African Americans and age 10 for Caucasians and Hispanics. The first sign is an enlargement of the testes. Pubic hair appears next, and then penis size increases.

A patient is able to flex his right arm forward without difficulty or pain but is unable to abduct his arm because of pain and muscle spasms. The nurse should suspect: a. Crepitation. b. Rotator cuff lesions. c. Dislocated shoulder. d. Rheumatoid arthritis.

ANS: B Rotator cuff lesions may limit range of motion and cause pain and muscle spasms during abduction, whereas forward flexion remains fairly normal. The other options are not correct.

When assessing the scrotum of a male patient, the nurse notices the presence of multiple firm, nontender, yellow 1-cm nodules. The nurse knows that these nodules are most likely: a. From urethritis. b. Sebaceous cysts. c. Subcutaneous plaques. d. From an inflammation of the epididymis.

ANS: B Sebaceous cysts are commonly found on the scrotum. These yellowish 1-cm nodules are firm, nontender, and often multiple. The other options are not correct.

In obtaining a health history on a 74-year-old patient, the nurse notes that he drinks alcohol daily and that he has noticed a tremor in his hands that affects his ability to hold things. With this information, what response should the nurse make? a. Does your family know you are drinking every day? b. Does the tremor change when you drink alcohol? c. We'll do some tests to see what is causing the tremor. d. You really shouldn't drink so much alcohol; it may be causing your tremor.

ANS: B Senile tremor is relieved by alcohol, although not a recommended treatment. The nurse should assess whether the person is abusing alcohol in an effort to relieve the tremor.

A 50-year-old woman calls the clinic because she has noticed some changes in her body and breasts and wonders if these changes could be attributable to the hormone replacement therapy (HRT) she started 3 months earlier. The nurse should tell her: a. HRT is at such a low dose that side effects are very unusual. b. HRT has several side effects, including fluid retention, breast tenderness, and vaginal bleeding. c. Vaginal bleeding with HRT is very unusual; I suggest you come into the clinic immediately to have this evaluated. d. It sounds as if your dose of estrogen is too high; I think you may need to decrease the amount you are taking and then call back in a week.

ANS: B Side effects of HRT include fluid retention, breast pain, and vaginal bleeding. The other responses are not correct.

The nurse places a key in the hand of a patient and he identifies it as a penny. What term would the nurse use to describe this finding? a. Extinction b. Astereognosis c. Graphesthesia d. Tactile discrimination

ANS: B Stereognosis is the persons ability to recognize objects by feeling their forms, sizes, and weights. Astereognosis is an inability to identify objects correctly, and it occurs in sensory cortex lesions. Tactile discrimination tests fine touch. Extinction tests the persons ability to feel sensations on both sides of the body at the same point.

A 52-year-old patient states that when she sneezes or coughs she wets herself a little. She is very concerned that something may be wrong with her. The nurse suspects that the problem is: a. Dysuria. b. Stress incontinence. c. Hematuria. d. Urge incontinence.

ANS: B Stress incontinence is involuntary urine loss with physical strain, sneezing, or coughing. Dysuria is pain or burning with urination. Hematuria is bleeding with urination. Urge incontinence is involuntary urine loss that occurs as a result of an overactive detrusor muscle in the bladder that contracts and causes an urgent need to void.

The nurse is performing an examination of the anus and rectum. Which of these statements is correct and important to remember during this examination? a. The rectum is approximately 8 cm long. b. The anorectal junction cannot be palpated. c. Above the anal canal, the rectum turns anteriorly. d. No sensory nerves are in the anal canal or rectum.

ANS: B The anal columns are folds of mucosa that extend vertically down from the rectum and end in the anorectal junction. This junction is not palpable but is visible on proctoscopy. The rectum is 12 cm long; just above the anal canal, the rectum dilates and turns posteriorly.

During an interview the patient states, I can feel this bump on the top of both of my shouldersit doesnt hurt but I am curious about what it might be. The nurse should tell the patient that it is his: a. Subacromial bursa. b. Acromion process. c. Glenohumeral joint. d. Greater tubercle of the humerus.

ANS: B The bump of the scapulas acromion process is felt at the very top of the shoulder. The other options are not correct.

During an examination, the nurse would expect the cervical os of a woman who has never had children to appear: a. Stellate. b. Small and round. c. As a horizontal irregular slit. d. Everted.

ANS: B The cervical os in a nulliparous woman is small and round. In the parous woman, it is a horizontal, irregular slit that also may show healed lacerations on the sides

During an internal examination, the nurse notices that the cervix bulges outside the introitus when the patient is asked to strain. The nurse will document this as: a. Uterine prolapse, graded first degree. b. Uterine prolapse, graded second degree. c. Uterine prolapse, graded third degree. d. A normal finding.

ANS: B The cervix should not be found to bulge into the vagina. Uterine prolapse is graded as follows: first degreethe cervix appears at the introitus with straining; second degreethe cervix bulges outside the introitus with straining; and third degreethe whole uterus protrudes, even without straining (essentially, the uterus is inside out).

When assessing a woman who is in her third trimester of pregnancy, the nurse looks for the classic symptoms associated with preeclampsia, which include: a. Edema, headaches, and seizures. b. Elevated blood pressure and proteinuria. c. Elevated liver enzymes and high platelet counts. d. Decreased blood pressure and edema.

ANS: B The classic symptoms of preeclampsia are hypertension and proteinuria. Headaches may occur with worsening symptoms, and seizures may occur if preeclampsia is left untreated and leads to eclampsia. A serious variant of preeclampsia, the hemolysis, elevated liver enzymes, low platelet count (HELLP) syndrome, is an ominous picture. Edema is a common occurrence in pregnancy.

Which statement is true regarding the recording of data from the history and physical examination? a. Use long, descriptive sentences to document findings. b. Record the data as soon as possible after the interview and physical examination. c. If the information is not documented, then it can be assumed that it was done as a standard of care. d. The examiner should avoid taking any notes during the history and examination because of the possibility of decreasing the rapport with the patient.

ANS: B The data from the history and physical examination should be recorded as soon after the event as possible. From a legal perspective, if it is not documented, then it was not done. Brief notes should be taken during the examination. When documenting, the nurse should use short, clear phrases and avoid redundant phrases and descriptions.

When the nurse is discussing sexuality and sexual issues with an adolescent, a permission statement helps convey that it is normal to think or feel a certain way. Which statement is the best example of a permission statement? a. It is okay that you have become sexually active. b. Girls your age often have questions about sexual activity. Do you have any questions? c. If it is okay with you, Id like to ask you some questions about your sexual history. d. Girls your age often engage in sexual activities. It is okay to tell me if you have had intercourse.

ANS: B The examiner should start with a permission statement such as, Girls your age often experience A permission statement conveys the idea that it is normal to think or feel a certain way, and implying that the topic is normal and unexceptional is important.

The external male genital structures include the: a. Testis. b. Scrotum. c. Epididymis. d. Vas deferens.

ANS: B The external male genital structures include the penis and scrotum. The testis, epididymis, and vas deferens are internal structures.

Which statement concerning the sphincters is correct? a. The internal sphincter is under voluntary control. b. The external sphincter is under voluntary control. c. Both sphincters remain slightly relaxed at all times. d. The internal sphincter surrounds the external sphincter.

ANS: B The external sphincter surrounds the internal sphincter but also has a small section overriding the tip of the internal sphincter at the opening. The external sphincter is under voluntary control. Except for the passing of feces and gas, the sphincters keep the anal canal tightly closed.

During an assessment of the CNs, the nurse finds the following: asymmetry when the patient smiles or frowns, uneven lifting of the eyebrows, sagging of the lower eyelids, and escape of air when the nurse presses against the right puffed cheek. This would indicate dysfunction of which of these CNs? a. Motor component of CN IV b. Motor component of CN VII c. Motor and sensory components of CN XI d. Motor component of CN X and sensory component of CN VII

ANS: B The findings listed reflect a dysfunction of the motor component of the facial nerve (CN VII).

The nurse is examining the hip area of a patient and palpates a flat depression on the upper, lateral side of the thigh when the patient is standing. The nurse interprets this finding as the: a. Ischial tuberosity. b. Greater trochanter. c. Iliac crest. d. Gluteus maximus muscle.

ANS: B The greater trochanter of the femur is palpated when the person is standing, and it appears as a flat depression on the upper lateral side of the thigh. The iliac crest is the upper part of the hip bone; the ischial tuberosity lies under the gluteus maximus muscle and is palpable when the hip is flexed; and the gluteus muscle is part of the buttocks.

Which statement is true regarding the complete physical assessment? a. The male genitalia should be examined in the supine position. b. The patient should be in the sitting position for examination of the head and neck. c. The vital signs, height, and weight should be obtained at the end of the examination. d. To promote consistency between patients, the examiner should not vary the order of the assessment.

ANS: B The head and neck should be examined in the sitting position to best palpate the thyroid and lymph nodes. The male patient should stand during an examination of the genitalia. Vital signs are measured early in the assessment. The sequence of the assessment may need to vary according to different patient situations.

Which statement concerning the areas of the brain is true? a. The cerebellum is the center for speech and emotions. b. The hypothalamus controls body temperature and regulates sleep. c. The basal ganglia are responsible for controlling voluntary movements. d. Motor pathways of the spinal cord and brainstem synapse in the thalamus.

ANS: B The hypothalamus is a vital area with many important functions: body temperature controller, sleep center, anterior and posterior pituitary gland regulator, and coordinator of autonomic nervous system activity and emotional status. The cerebellum controls motor coordination, equilibrium, and balance. The basal ganglia control autonomic movements of the body. The motor pathways of the spinal cord synapse in various areas of the spinal cord, not in the thalamus.

A mother of a 1-month-old infant asks the nurse why it takes so long for infants to learn to roll over. The nurse knows that the reason for this is: a. A demyelinating process must be occurring with her infant. b. Myelin is needed to conduct the impulses, and the neurons of a newborn are not yet myelinated. c. The cerebral cortex is not fully developed; therefore, control over motor function gradually occurs. d. The spinal cord is controlling the movement because the cerebellum is not yet fully developed.

ANS: B The infants sensory and motor development proceeds along with the gradual acquisition of myelin, which is needed to conduct most impulses. Very little cortical control exists, and the neurons are not yet myelinated. The other responses are not correct.

The nurse is assessing a patients ischial tuberosity. To palpate the ischial tuberosity, the nurse knows that it is best to have the patient: a. Standing. b. Flexing the hip. c. Flexing the knee. d. Lying in the supine position.

ANS: B The ischial tuberosity lies under the gluteus maximus muscle and is palpable when the hip is flexed. The other options are not correct

The two parts of the nervous system are the: a. Motor and sensory. b. Central and peripheral. c. Peripheral and autonomic. d. Hypothalamus and cerebral.

ANS: B The nervous system can be divided into two partscentral and peripheral. The central nervous system includes the brain and spinal cord. The peripheral nervous system includes the 12 pairs of cranial nerves (CNs), the 31 pairs of spinal nerves, and all of their branches.

Which characteristic of the prostate gland would the nurse recognize as an abnormal finding while palpating the prostate gland through the rectum? a. Palpable central groove b. Tenderness to palpation c. Heart shaped d. Elastic and rubbery consistency

ANS: B The normal prostate gland should feel smooth, elastic, and rubbery; slightly movable; heart-shaped with a palpable central groove; and not be tender to palpation.

During the examination portion of a patients visit, she will be in lithotomy position. Which statement reflects some things that the nurse can do to make this position more comfortable for her? a. Ask her to place her hands and arms over her head. b. Elevate her head and shoulders to maintain eye contact. c. Allow her to choose to have her feet in the stirrups or have them resting side by side on the edge of the table. d. Allow her to keep her buttocks approximately 6 inches from the edge of the table to prevent her from feeling as if she will fall off.

ANS: B The nurse should elevate her head and shoulders to maintain eye contact. The patients arms should be placed at her sides or across the chest. Placing her hands and arms over her head only tightens the abdominal muscles. The feet should be placed into the stirrups, knees apart, and buttocks at the edge of the examining table. The stirrups are placed so that the legs are not abducted too far.

During a health history interview, a 38-year-old woman shares that she is thinking about having another baby. The nurse knows which statement to be true regarding pregnancy after 35 years of age? a. Fertility does not start to decline until age 40 years. b. Occurrence of Down syndrome is significantly more frequent after the age of 35 years. c. Genetic counseling and prenatal screening are not routine until after age 40 years. d. Women older than 35 years who are pregnant have the same rate of pregnancy-related complications as those who are younger than 35 years.

ANS: B The risk of Down syndrome increases as the woman ages, from approximately 1 in 1250 at age 25 years to 1 in 400 at age 35 years. Fertility declines with advancing maternal age. Women 35 years and older or with a history of a genetic abnormality are offered genetic counseling and the options of prenatal diagnostic screening tests. Because the incidence of chronic diseases increases with age, women older than 35 years who are pregnant more often have medical complications such as diabetes, obesity, and hypertension.

During the neurologic assessment of a healthy 35-year-old patient, the nurse asks him to relax his muscles completely. The nurse then moves each extremity through full range of motion. Which of these results would the nurse expect to find? a. Firm, rigid resistance to movement b. Mild, even resistance to movement c. Hypotonic muscles as a result of total relaxation d. Slight pain with some directions of movement

ANS: B Tone is the normal degree of tension (contraction) in voluntarily relaxed muscles. It shows a mild resistance to passive stretching. Normally, the nurse will notice a mild, even resistance to movement. The other responses are not correct.

An 85-year-old man has come in for a physical examination, and the nurse notices that he uses a cane. When documenting general appearance, the nurse should document this information under the section that covers: a. Posture. b. Mobility. c. Mood and affect. d. Physical deformity.

ANS: B Use of assistive devices would be documented under the mobility section. The other responses are all other categories of the general appearance section of the health history.

During an examination of an aging man, the nurse recognizes that normal changes to expect would be: a. Change in scrotal color. b. Decrease in the size of the penis. c. Enlargement of the testes and scrotum. d. Increase in the number of rugae over the scrotal sac.

ANS: B When assessing the genitals of an older man, the nurse may notice thinner, graying pubic hair and a decrease in the size of the penis. The size of the testes may be decreased, they may feel less firm, and the scrotal sac is pendulous with less rugae. No change in scrotal color is observed.

A patient calls the clinic for instructions before having a Papanicolaou (Pap) smear. The most appropriate instructions from the nurse are: a. If you are menstruating, please use pads to avoid placing anything into the vagina. b. Avoid intercourse, inserting anything into the vagina, or douching within 24 hours of your appointment. c. If you suspect that you have a vaginal infection, please gather a sample of the discharge to bring with you. d. We would like you to use a mild saline douche before your examination. You may pick this up in our office.

ANS: B When instructing a patient before Pap smear is obtained, the nurse should follow these guidelines: Do not obtain during the womans menses or if a heavy infectious discharge is present. Instruct the woman not to douche, have intercourse, or put anything into the vagina within 24 hours before collecting the specimens. Any specimens will be obtained during the visit, not beforehand.

The nurse is assessing the joints of a woman who has stated, I have a long family history of arthritis, and my joints hurt. The nurse suspects that she has osteoarthritis. Which of these are symptoms of osteoarthritis? Select all that apply. a. Symmetric joint involvement b. Asymmetric joint involvement c. Pain with motion of affected joints d. Affected joints are swollen with hard, bony protuberances e. Affected joints may have heat, redness, and swelling

ANS: B, C, D In osteoarthritis, asymmetric joint involvement commonly affects hands, knees, hips, and lumbar and cervical segments of the spine. Affected joints have stiffness, swelling with hard bony protuberances, pain with motion, and limitation of motion. The other options reflect the signs of rheumatoid arthritis.

A 69-year-old patient has been admitted to an adult psychiatric unit because his wife thinks he is getting more and more confused. He laughs when he is found to be forgetful, saying Im just getting old! After the nurse completes a thorough neurologic assessment, which findings would be indicative of Alzheimer disease? Select all that apply. a. Occasionally forgetting names or appointments b. Difficulty performing familiar tasks, such as placing a telephone call c. Misplacing items, such as putting dish soap in the refrigerator d. Sometimes having trouble finding the right word e. Rapid mood swings, from calm to tears, for no apparent reason f. Getting lost in ones own neighborhood

ANS: B, C, E, F Difficulty performing familiar tasks, misplacing items, rapid mood swings, and getting lost in ones own neighborhood can be warning signs of Alzheimer disease. Occasionally forgetting names or appointments, and sometimes having trouble finding the right word are part of normal aging.

The nurse is palpating an ovarian mass during an internal examination of a 63-year-old woman. Which findings of the masss characteristics would suggest the presence of an ovarian cyst? Select all that apply. a. Heavy and solid b. Mobile and fluctuant c. Mobile and solid d. Fixed e. Smooth and round f. Poorly defined

ANS: B, E An ovarian cyst (fluctuant ovarian mass) is usually asymptomatic and would feel like a smooth, round, fluctuant, mobile, nontender mass on the ovary. A mass that is heavy, solid, fixed, and poorly defined suggests malignancy. A benign mass may feel mobile and solid.

A patient has a severed spinal nerve as a result of trauma. Which statement is true in this situation? a. Because there are 31 pairs of spinal nerves, no effect results if only one nerve is severed. b. The dermatome served by this nerve will no longer experience any sensation. c. The adjacent spinal nerves will continue to carry sensations for the dermatome served by the severed nerve. d. A severed spinal nerve will only affect motor function of the patient because spinal nerves have no sensory component.

ANS: C A dermatome is a circumscribed skin area that is primarily supplied from one spinal cord segment through a particular spinal nerve. The dermatomes overlap, which is a form of biologic insurance; that is, if one nerve is severed, then most of the sensations can be transmitted by the spinal nerve above and the spinal nerve below the severed nerve.

A teenage girl has arrived complaining of pain in her left wrist. She was playing basketball when she fell and landed on her left hand. The nurse examines her hand and would expect a fracture if the girl complains of a: a. Dull ache. b. Deep pain in her wrist. c. Sharp pain that increases with movement. d. Dull throbbing pain that increases with rest.

ANS: C A fracture causes sharp pain that increases with movement. The other types of pain do not occur with a fracture.

During the examination of a patient, the nurse notices that the patient has several small, flat macules on the posterior portion of her thorax. These macules are less than 1 cm wide. Another name for these macules is: a. Warts. b. Bullae. c. Freckles. d. Papules.

ANS: C A macule is solely a lesion with color change, flat and circumscribed, less than 1 cm. Macules are also known as freckles

The nurse is palpating the fundus of a pregnant woman. Which statement about palpation of the fundus is true? a. The fundus should be hard and slightly tender to palpation during the first trimester. b. Fetal movement may not be felt by the examiner until the end of the second trimester. c. After 20 weeks gestation, the number of centimeters should approximate the number of weeks gestation. d. Fundal height is usually less than the number of weeks gestation, unless an abnormal condition such as excessive amniotic fluid is present.

ANS: C After 20 weeks gestation, the number of centimeters should approximate the number of weeks gestation. In addition, at 20 weeks gestation, the examiner may be able to feel fetal movement and the head can be balloted.

The nurse is assessing a 1-week-old infant and is testing his muscle strength. The nurse lifts the infant with hands under the axillae and notices that the infant starts to slip between the hands. The nurse should: a. Suspect a fractured clavicle. b. Suspect that the infant may have a deformity of the spine. c. Suspect that the infant may have weakness of the shoulder muscles. d. Conclude that this is a normal finding because the musculature of an infant at this age is undeveloped.

ANS: C An infant who starts to slip between the nurses hands shows weakness of the shoulder muscles. An infant with normal muscle strength wedges securely between the nurses hands. The other responses are not correct.

The nurse has just recorded a positive iliopsoas test on a patient who has abdominal pain. This test is used to confirm a(n): a. Inflamed liver. b. Perforated spleen. c. Perforated appendix. d. Enlarged gallbladder.

ANS: C An inflamed or perforated appendix irritates the iliopsoas muscle, producing pain in the RLQ.

When assessing a patients general appearance, the nurse should include which question? a. Is the patients muscle strength equal in both arms? b. Is ptosis or facial droop present? c. Does the patient appropriately respond to questions? d. Are the pupils equal in reaction and size?

ANS: C Assessing whether the patient appropriately responds to questions is a component of an assessment of the patients general appearance. The other answers reflect components of the neurologic examination.

When assessing the neurologic system of a hospitalized patient during morning rounds, the nurse should include which of these during the assessment? a. Blood pressure b. Patients rating of pain on a scale of 1 to 10 c. Patients ability to communicate d. Patients personal hygiene level

ANS: C Assessment of a patients ability to communicate is part of the neurologic assessment. Blood pressure and pain rating are measurements, and personal hygiene is assessed under general appearance.

When taking the health history on a patient with a seizure disorder, the nurse assesses whether the patient has an aura. Which of these would be the best question for obtaining this information? a. Does your muscle tone seem tense or limp? b. After the seizure, do you spend a lot of time sleeping? c. Do you have any warning sign before your seizure starts? d. Do you experience any color change or incontinence during the seizure?

ANS: C Aura is a subjective sensation that precedes a seizure; it could be auditory, visual, or motor. The other questions do not solicit information about an aura.

Generally, the changes normally associated with menopause occur because the cells in the reproductive tract are: a. Aging. b. Becoming fibrous. c. Estrogen dependent. d. Able to respond to estrogen.

ANS: C Because cells in the reproductive tract are estrogen dependent, decreased estrogen levels during menopause bring dramatic physical changes. The other options are not correct.

The nurse auscultates a functional systolic murmur, grade II/IV, on a woman in week 30 of her pregnancy. The remainder of her physical assessment is within normal limits. The nurse would: a. Consider this finding abnormal, and refer her for additional consultation. b. Ask the woman to run briefly in place and then assess for an increase in intensity of the murmur. c. Know that this finding is normal and is a result of the increase in blood volume during pregnancy. d. Ask the woman to restrict her activities and return to the clinic in 1 week for re-evaluation.

ANS: C Because of the increase in blood volume, a functional systolic murmur, grade II/IV or less, can be heard in 95% of pregnant women. The other actions are not appropriate.

A patient who is visiting the clinic complains of having stomach pains for 2 weeks and describes his stools as being soft and black for approximately the last 10 days. He denies taking any medications. The nurse is aware that these symptoms are mostly indicative of: a. Excessive fat caused by malabsorption. b. Increased iron intake, resulting from a change in diet. c. Occult blood, resulting from gastrointestinal bleeding. d. Absent bile pigment from liver problems.

ANS: C Black stools may be tarry as a result of occult blood (melena) from gastrointestinal bleeding or nontarry from ingestion of iron medications (not diet). Excessive fat causes the stool to become frothy. The absence of bile pigment causes clay-colored stools.

A patient who has had rheumatoid arthritis for years comes to the clinic to ask about changes in her fingers. The nurse will assess for signs of what problems? a. Heberden nodes b. Bouchard nodules c. Swan-neck deformities d. Dupuytren contractures

ANS: C Changes in the fingers caused by chronic rheumatoid arthritis include swan-neck and boutonniere deformities. Heberden nodes and Bouchard nodules are associated with osteoarthritis. Dupuytren contractures of the digits occur because of chronic hyperplasia of the palmar fascia

A patient is unable to shrug her shoulders against the nurses resistant hands. What cranial nerve is involved with successful shoulder shrugging? a. VII b. IX c. XI d. XII

ANS: C Cranial nerve XI enables the patient to shrug her shoulders against resistance.

The nurse is performing a genitourinary assessment on a 50-year-old obese male laborer. On examination, the nurse notices a painless round swelling close to the pubis in the area of the internal inguinal ring that is easily reduced when the individual is supine. These findings are most consistent with a(n) ______ hernia. a. Scrotal b. Femoral c. Direct inguinal d. Indirect inguinal

ANS: C Direct inguinal hernias occur most often in men over the age of 40 years. It is an acquired weakness brought on by heavy lifting, obesity, chronic cough, or ascites. The direct inguinal hernia is usually a painless, round swelling close to the pubis in the area of the internal inguinal ring that is easily reduced when the individual is supine.

A man who was found wandering in a park at 2 AM has been brought to the emergency department for an examination; he said he fell and hit his head. During the examination, the nurse asks him to use his index finger to touch the nurses finger, then his own nose, then the nurses finger again (which has been moved to a different location). The patient is clumsy, unable to follow the instructions, and overshoots the mark, missing the finger. The nurse should suspect which of the following? a. Cerebral injury b. Cerebrovascular accident c. Acute alcohol intoxication d. Peripheral neuropathy

ANS: C During the finger-to-finger test, if the person has clumsy movement with overshooting the mark, either a cerebellar disorder or acute alcohol intoxication should be suspected. The persons movements should be smooth and accurate. The other options are not correct.

During the examination of a woman in her second trimester of pregnancy, the nurse notices the presence of a small amount of yellow drainage from the nipples. The nurse knows that this is: a. An indication that the woman's milk is coming in. b. A sign of possible breast cancer in a pregnant woman. c. Most likely colostrum and considered a normal finding at this stage of the pregnancy. d. Too early in the pregnancy for lactation to begin and refers the woman to a specialist.

ANS: C During the second trimester, colostrum, the precursor of milk, may be expressed from the nipples. Colostrum is yellow and contains more minerals and protein but less sugar and fat than mature milk. The other options are incorrect.

A woman who is 8 weeks pregnant is visiting the clinic for a checkup. Her systolic blood pressure is 30 mm Hg higher than her pre-pregnancy systolic blood pressure. The nurse should: a. Consider this a normal finding. b. Expect the blood pressure to decrease as the estrogen levels increase throughout the pregnancy. c. Consider this an abnormal finding because blood pressure is typically lower at this point in the pregnancy. d. Recommend that she decrease her salt intake in an attempt to decrease her peripheral vascular resistance.

ANS: C During the seventh gestational week, blood pressure begins to drop as a result of falling peripheral vascular resistance. Early in the first trimester, blood pressure values are similar to those of prepregnancy measurements. In this case, the womans blood pressure is higher than it should be.

During an examination, the nurse notices that a patients legs turn white when they are raised above the patients head. The nurse should suspect: a. Lymphedema. b. Raynaud disease. c. Chronic arterial insufficiency. d. Chronic venous insufficiency.

ANS: C Elevational pallor (striking) indicates arterial insufficiency

A mother brings her newborn baby boy in for a checkup; she tells the nurse that he does not seem to be moving his right arm as much as his left and that he seems to have pain when she lifts him up under the arms. The nurse suspects a fractured clavicle and would observe for: a. Negative Allis test. b. Positive Ortolani sign. c. Limited range of motion during the Moro reflex. d. Limited range of motion during Lasgue test.

ANS: C For a fractured clavicle, the nurse should observe for limited arm range of motion and unilateral response to the Moro reflex. The other tests are not appropriate for this type of fracture.

The nurse suspects that a patient has carpal tunnel syndrome and wants to perform the Phalen test. To perform this test, the nurse should instruct the patient to: a. Dorsiflex the foot. b. Plantarflex the foot. c. Hold both hands back to back while flexing the wrists 90 degrees for 60 seconds. d. Hyperextend the wrists with the palmar surface of both hands touching, and wait for 60 seconds.

ANS: C For the Phalen test, the nurse should ask the person to hold both hands back to back while flexing the wrists 90 degrees. Acute flexion of the wrist for 60 seconds produces no symptoms in the normal hand. The Phalen test reproduces numbness and burning in a person with carpal tunnel syndrome. The other actions are not correct when testing for carpal tunnel syndrome.

Which of these is included in an assessment of general appearance? a. Height b. Weight c. Skin color d. Vital signs

ANS: C General appearance includes items such as level of consciousness, skin color, nutritional status, posture, mobility, facial expression, mood and affect, speech, hearing, and personal hygiene. Height, weight, and vital signs are considered measurements.

During a genital examination, the nurse notices that a male patient has clusters of small vesicles on the glans, surrounded by erythema. The nurse recognizes that these lesions are: a. Peyronie disease. b. Genital warts. c. Genital herpes. d. Syphilitic cancer.

ANS: C Genital herpes, or herpes simplex virus 2 (HSV-2), infections are indicated with clusters of small vesicles with surrounding erythema, which are often painful and erupt on the glans or foreskin.

An accessory glandular structure for the male genital organs is the: a. Testis. b. Scrotum. c. Prostate. d. Vas deferens.

ANS: C Glandular structures accessory to the male genital organs are the prostate, seminal vesicles, and bulbourethral glands.

The nurse should wear gloves for which of these examinations? a. Measuring vital signs b. Palpation of the sinuses c. Palpation of the mouth and tongue d. Inspection of the eye with an ophthalmoscope

ANS: C Gloves should be worn when the examiner is exposed to the patients body fluids.

A patient has had three pregnancies and two live births. The nurse would record this information as grav _____, para _____, AB _____. a. 2; 2; 1 b. 3; 2; 0 c. 3; 2; 1 d. 3; 3; 1

ANS: C Gravida (grav) is the number of pregnancies. Para is the number of births. Abortions are interrupted pregnancies, including elective abortions and spontaneous miscarriages.

During an external genitalia examination of a woman, the nurse notices several lesions around the vulva. The lesions are pink, moist, soft, and pointed papules. The patient states that she is not aware of any problems in that area. The nurse recognizes that these lesions may be: a. Syphilitic chancre. b. Herpes simplex virus type 2 (herpes genitalis). c. HPV or genital warts. d. Pediculosis pubis (crab lice).

ANS: C HPV lesions are painless, warty growths that the woman may not notice. Lesions are pink or flesh colored, soft, pointed, moist, warty papules that occur in single or multiple cauliflower-like patches around the vulva, introitus, anus, vagina, or cervix. Herpetic lesions are painful clusters of small, shallow vesicles with surrounding erythema. Syphilitic chancres begin as a solitary silvery papule that erodes into a red, round or oval superficial ulcer with a yellowish discharge. Pediculosis pubis causes severe perineal itching and excoriations and erythematous areas

During the interview with a female patient, the nurse gathers data that indicate the patient is perimenopausal. Which of these statements made by this patient leads to this conclusion? a. I have noticed that my muscles ache at night when I go to bed. b. I will be very happy when I can stop worrying about having a period. c. I have been noticing that I sweat a lot more than I used to, especially at night. d. I have only been pregnant twice, but both times I had breast tenderness as my first symptom.

ANS: C Hormone shifts occur during the perimenopausal period, and associated symptoms of menopause may occur, such as hot flashes, night sweats, numbness and tingling, headache, palpitations, drenching sweats, mood swings, vaginal dryness, and itching. The other responses are not correct.

While recording in a patients medical record, the nurse notices that a patients Hematest results are positive. This finding means that there is(are): a. Crystals in his urine. b. Parasites in his stool. c. Occult blood in his stool. d. Bacteria in his sputum.

ANS: C If a stool is Hematest positive, then it indicates the presence of occult blood

A 2-month-old uncircumcised infant has been brought to the clinic for a well-baby checkup. How would the nurse proceed with the genital examination? a. Eliciting the cremasteric reflex is recommended. b. The glans is assessed for redness or lesions. c. Retracting the foreskin should be avoided until the infant is 3 months old. d. Any dirt or smegma that has collected under the foreskin should be noted.

ANS: C If uncircumcised, then the foreskin is normally tight during the first 3 months and should not be retracted because of the risk of tearing the membrane attaching the foreskin to the shaft. The other options are not correct.

During a morning assessment, the nurse notices that a patients urine output is below the expected amount. What should the nurse do next? a. Obtain an order for a Foley catheter. b. Obtain an order for a straight catheter. c. Perform a bladder scan test. d. Refer the patient to an urologist.

ANS: C If urine output is below the expected value, then the nurse should perform a bladder scan according to institutional policy to check for retention.

During an examination of an aging man, the nurse recognizes that normal changes to expect would be: a. Enlarged scrotal sac. b. Increased pubic hair. c. Decreased penis size. d. Increased rugae over the scrotum.

ANS: C In the aging man, the amount of pubic hair decreases, the penis size decreases, and the rugae over the scrotal sac decreases. The scrotal sac does not enlarge.

During an examination, the nurse asks a patient to bend forward from the waist and notices that the patient has lateral tilting. When his leg is raised straight up, the patient complains of a pain going down his buttock into his leg. The nurse suspects: a. Scoliosis. b. Meniscus tear. c. Herniated nucleus pulposus. d. Spasm of paravertebral muscles.

ANS: C Lateral tilting and sciatic pain with straight leg raising are findings that occur with a herniated nucleus pulposus. The other options are not correct.

The nurse is explaining the mechanism of the growth of long bones to a mother of a toddler. Where does lengthening of the bones occur? a. Bursa b. Calcaneus c. Epiphyses d. Tuberosities

ANS: C Lengthening occurs at the epiphyses, or growth plates. The other options are not correct.

The nurse is performing a neurologic assessment on a 41-year-old woman with a history of diabetes. When testing her ability to feel the vibrations of a tuning fork, the nurse notices that the patient is unable to feel vibrations on the great toe or ankle bilaterally, but she is able to feel vibrations on both patellae. Given this information, what would the nurse suspect? a. Hyperalgesia b. Hyperesthesia c. Peripheral neuropathy d. Lesion of sensory cortex

ANS: C Loss of vibration sense occurs with peripheral neuropathy (e.g., diabetes and alcoholism). Peripheral neuropathy is worse at the feet and gradually improves as the examiner moves up the leg, as opposed to a specific nerve lesion, which has a clear zone of deficit for its dermatome. The other responses are incorrect.

The nurse is aware of which statement to be true regarding the incidence of testicular cancer? a. Testicular cancer is the most common cancer in men aged 30 to 50 years. b. The early symptoms of testicular cancer are pain and induration. c. Men with a history of cryptorchidism are at the greatest risk for the development of testicular cancer. d. The cure rate for testicular cancer is low.

ANS: C Men with undescended testicles (cryptorchidism) are at the greatest risk for the development of testicular cancer. The overall incidence of testicular cancer is rare. Although testicular cancer has no early symptoms, when detected early and treated before metastasizing, the cure rate is almost 100%.

A 2-year-old boy has been diagnosed with physiologic cryptorchidism. Considering this diagnosis, during assessment the nurse will most likely observe: a. Testes that are hard and painful to palpation. b. Atrophic scrotum and a bilateral absence of the testis. c. Absence of the testis in the scrotum, but the testis can be milked down. d. Testes that migrate into the abdomen when the child squats or sits cross-legged.

ANS: C Migratory testes (physiologic cryptorchidism) are common because of the strength of the cremasteric reflex and the small mass of the prepubertal testes. The affected side has a normally developed scrotum and the testis can be milked down. The other responses are not correct.

A patient is being assessed for range-of-joint movement. The nurse asks him to move his arm in toward the center of his body. This movement is called: a. Flexion. b. Abduction. c. Adduction. d. Extension.

ANS: C Moving a limb toward the midline of the body is called adduction; moving a limb away from the midline of the body is called abduction. Flexion is bending a limb at a joint; and extension is straightening a limb at a joint.

The nurse is examining a 35-year-old female patient. During the health history, the nurse notices that she has had two term pregnancies, and both babies were delivered vaginally. During the internal examination, the nurse observes that the cervical os is a horizontal slit with some healed lacerations and that the cervix has some nabothian cysts that are small, smooth, and yellow. In addition, the nurse notices that the cervical surface is granular and red, especially around the os. Finally, the nurse notices the presence of stringy, opaque, odorless secretions. Which of these findings are abnormal? a. Nabothian cysts are present. b. The cervical os is a horizontal slit. c. The cervical surface is granular and red. d. Stringy and opaque secretions are present.

ANS: C Normal findings: Nabothian cysts may be present on the cervix after childbirth. The cervical os is a horizontal, irregular slit in the parous woman. Secretions vary according to the day of the menstrual cycle, and may be clear and thin or thick, opaque, and stringy. The surface is normally smooth, but cervical eversion, or ectropion, may occur where the endocervical canal is rolled out. Abnormal finding: The cervical surface should not be reddened or granular, which may indicate a lesion.

When performing the bimanual examination, the nurse notices that the cervix feels smooth and firm, is round, and is fixed in place (does not move). When cervical palpation is performed, the patient complains of some pain. The nurses interpretation of these results should be which of these? a. These findings are all within normal limits. b. Cervical consistency should be soft and velvety not firm. c. The cervix should move when palpated; an immobile cervix may indicate malignancy. d. Pain may occur during palpation of the cervix.

ANS: C Normally, the cervix feels smooth and firm, similar to the consistency of the tip of the nose. It softens and feels velvety at 5 to 6 weeks of pregnancy (Goodell sign). The cervix should be evenly rounded. With a finger on either side, the examiner should be able to move the cervix gently from side to side, and doing so should produce no pain for the patient. Hardness of the cervix may occur with malignancy. Immobility may occur with malignancy, and pain may occur with inflammation or ectopic pregnancy.

When the nurse is interviewing a preadolescent girl, which opening question would be least threatening? a. Do you have any questions about growing up? b. What has your mother told you about growing up? c. When did you notice that your body was changing? d. I remember being very scared when I got my period. How do you think you'll feel?

ANS: C Open-ended questions such as, When did you ? rather than Do you ? should be asked. Open-ended questions are less threatening because they imply that the topic is normal and unexceptional.

A 14-year-old boy who has been diagnosed with Osgood-Schlatter disease reports painful swelling just below the knee for the past 5 months. Which response by the nurse is appropriate? a. If these symptoms persist, you may need arthroscopic surgery. b. You are experiencing degeneration of your knee, which may not resolve. c. Your disease is due to repeated stress on the patellar tendon. It is usually self-limited, and your symptoms should resolve with rest. d. Increasing your activity and performing knee-strengthening exercises will help decrease the inflammation and maintain mobility in the knee.

ANS: C Osgood-Schlatter disease is a painful swelling of the tibial tubercle just below the knee and most likely due to repeated stress on the patellar tendon. It is usually self-limited, occurring during rapid growth and most often in boys. The symptoms resolve with rest. The other responses are not appropriate.

During a bimanual examination, the nurse detects a solid tumor on the ovary that is heavy and fixed, with a poorly defined mass. This finding is suggestive of: a. Ovarian cyst. b. Endometriosis. c. Ovarian cancer. d. Ectopic pregnancy.

ANS: C Ovarian tumors that are solid, heavy, and fixed, with poorly defined mass are suggestive of malignancy. Benign masses may feel mobile and solid. An ovarian cyst may feel smooth, round, fluctuant, mobile, and nontender. With an ectopic pregnancy, the examiner may feel a palpable, tender pelvic mass that is solid, mobile, and unilateral. Endometriosis may have masses (in various locations in the pelvic area) that are small, firm, nodular, and tender to palpation, with enlarged ovaries.

A patient tells the nurse, Sometimes I wake up at night and I have real trouble breathing. I have to sit up in bed to get a good breath. When documenting this information, the nurse would note: a. Orthopnea. b. Acute emphysema. c. Paroxysmal nocturnal dyspnea. d. Acute shortness of breath episode.

ANS: C Paroxysmal nocturnal dyspnea occurs when the patient awakens from sleep with shortness of breath and needs to be upright to achieve comfort

The nurse is teaching a class on preventing osteoporosis to a group of perimenopausal women. Which of these actions is the best way to prevent or delay bone loss in this group? a. Taking calcium and vitamin D supplements b. Taking medications to prevent osteoporosis c. Performing physical activity, such as fast walking d. Assessing bone density annually

ANS: C Physical activity, such as fast walking, delays or prevents bone loss in perimenopausal women. The faster the pace of walking, the higher the preventive effect is on the risk of hip fracture. The other options are not correct.

During a neonatal examination, the nurse notices that the newborn infant has six toes. This finding is documented as: a. Unidactyly. b. Syndactyly. c. Polydactyly. d. Multidactyly.

ANS: C Polydactyly is the presence of extra fingers or toes. Syndactyly is webbing between adjacent fingers or toes. The other terms are not correct.

A patient who is 20 weeks pregnant tells the nurse that she feels more shortness of breath as her pregnancy progresses. The nurse recognizes which statement to be true? a. High levels of estrogen cause shortness of breath. b. Feelings of shortness of breath are abnormal during pregnancy. c. Hormones of pregnancy cause an increased respiratory effort. d. The patient should get more exercise in an attempt to increase her respiratory reserve.

ANS: C Progesterone and estrogen cause an increase in respiratory effort during pregnancy by increasing tidal volume. Increased tidal volume causes a slight drop in partial pressure of arterial carbon dioxide (PaCO2), causing the woman to have dyspnea occasionally. The other options are not correct.

During the interview, a patient reveals that she has some vaginal discharge. She is worried that it may be a sexually transmitted infection. The nurses most appropriate response to this would be: a. Oh, don't worry. Some cyclic vaginal discharge is normal. b. Have you been engaging in unprotected sexual intercourse? c. I'd like some information about the discharge. What color is it? d. Have you had any urinary incontinence associated with the discharge?

ANS: C Questions that help the patient reveal more information about her symptoms should be asked in a nonthreatening manner. Asking about the amount, color, and odor of the vaginal discharge provides the opportunity for further assessment. Normal vaginal discharge is small, clear or cloudy, and always nonirritating.

A patient is complaining of pain in his joints that is worse in the morning, better after he moves around for a while, and then gets worse again if he sits for long periods. The nurse should assess for other signs of what problem? a. Tendinitis b. Osteoarthritis c. Rheumatoid arthritis d. Intermittent claudication

ANS: C Rheumatoid arthritis is worse in the morning when a person arises. Movement increases most joint pain, except the pain with rheumatoid arthritis, which decreases with movement. The other options are not correct.

A woman states that 2 weeks ago she had a urinary tract infection that was treated with an antibiotic. As a part of the interview, the nurse should ask, Have you noticed any: a. Changes in your urination patterns? b. Excessive vaginal bleeding? c. Unusual vaginal discharge or itching? d. Changes in your desire for intercourse?

ANS: C Several medications may increase the risk of vaginitis. Broad-spectrum antibiotics alter the balance of normal flora, which may lead to the development of vaginitis. The other questions are not appropriate.

A patient is unable to perform rapid alternating movements such as rapidly patting her knees. The nurse should document this inability as: a. Ataxia. b. Astereognosis. c. Presence of dysdiadochokinesia. d. Loss of kinesthesia.

ANS: C Slow clumsy movements and the inability to perform rapid alternating movements occur with cerebellar disease. The condition is termed dysdiadochokinesia. Ataxia is an uncoordinated or unsteady gait. Astereognosis is the inability to identify an object by feeling it. Kinesthesia is the persons ability to perceive passive movement of the extremities or the loss of position sense.

During an assessment of an 80-year-old patient, the nurse notices the following: an inability to identify vibrations at her ankle and to identify the position of her big toe, a slower and more deliberate gait, and a slightly impaired tactile sensation. All other neurologic findings are normal. The nurse should interpret that these findings indicate: a. CN dysfunction. b. Lesion in the cerebral cortex. c. Normal changes attributable to aging. d. Demyelination of nerves attributable to a lesion.

ANS: C Some aging adults show a slower response to requests, especially for those calling for coordination of movements. The findings listed are normal in the absence of other significant abnormal findings. The other responses are incorrect.

The nurse is caring for a patient who has just had neurosurgery. To assess for increased intracranial pressure, what would the nurse include in the assessment? a. CNs, motor function, and sensory function b. Deep tendon reflexes, vital signs, and coordinated movements c. Level of consciousness, motor function, pupillary response, and vital signs d. Mental status, deep tendon reflexes, sensory function, and pupillary response

ANS: C Some hospitalized persons have head trauma or a neurologic deficit from a systemic disease process. These people must be closely monitored for any improvement or deterioration in neurologic status and for any indication of increasing intracranial pressure. The nurse should use an abbreviation of the neurologic examination in the following sequence: level of consciousness, motor function, pupillary response, and vital signs.

The nurse notices that a patient has had a pale, yellow, greasy stool, or steatorrhea, and recalls that this is caused by: a. Occult bleeding. b. Absent bile pigment. c. Increased fat content. d. Ingestion of bismuth preparations.

ANS: C Steatorrhea (pale, yellow, greasy stool) is caused by increased fat content in the stools, as in malabsorption syndrome. Occult bleeding and ingestion of bismuth products cause a black stool, and absent bile pigment causes a gray-tan stool.

A 45-year-old mother of two children is seen at the clinic for complaints of losing my urine when I sneeze. The nurse documents that she is experiencing: a. Urinary frequency. b. Enuresis. c. Stress incontinence. d. Urge incontinence.

ANS: C Stress incontinence is involuntary urine loss with physical strain, sneezing, or coughing that occurs as a result to weakness of the pelvic floor. Urinary frequency is urinating more times than usual (more than five to six times per day). Enuresis is involuntary passage of urine at night after age 5 to 6 years (bed wetting). Urge incontinence is involuntary urine loss from overactive detrusor muscle in the bladder. It contracts, causing an urgent need to void.

When the nurse is performing a testicular examination on a 25-year-old man, which finding is considered normal? a. Nontender subcutaneous plaques b. Scrotal area that is dry, scaly, and nodular c. Testes that feel oval and movable and are slightly sensitive to compression d. Single, hard, circumscribed, movable mass, less than 1 cm under the surface of the testes

ANS: C Testes normally feel oval, firm and rubbery, smooth, and bilaterally equal and are freely movable and slightly tender to moderate pressure. The scrotal skin should not be dry, scaly, or nodular or contain subcutaneous plaques. Any mass would be an abnormal finding.

If the nurse records the results to the Hirschberg test, the nurse has: a. Tested the patellar reflex. b. Assessed for appendicitis. c. Tested the corneal light reflex. d. Assessed for thrombophlebitis.

ANS: C The Hirschberg test assesses the corneal light reflex

The nurse will measure a patients near vision with which tool? a. Snellen eye chart with letters b. Snellen E chart c. Jaeger card d. Ophthalmoscope

ANS: C The Jaeger card is used to measure near vision

While assessing a 7-month-old infant, the nurse makes a loud noise and notices the following response: abduction and flexion of the arms and legs; fanning of the fingers, and curling of the index finger and thumb in a C position, followed by the infant bringing in the arms and legs to the body. What does the nurse know about this response? a. This response could indicate brachial nerve palsy. b. This reaction is an expected startle response at this age. c. This reflex should have disappeared between 1 and 4 months of age. d. This response is normal as long as the movements are bilaterally symmetric.

ANS: C The Moro reflex is present at birth and usually disappears at 1 to 4 months. Absence of the Moro reflex in the newborn or its persistence after 5 months of age indicates severe central nervous system injury. The other responses are incorrect.

The articulation of the mandible and the temporal bone is known as the: a. Intervertebral foramen. b. Condyle of the mandible. c. Temporomandibular joint. d. Zygomatic arch of the temporal bone.

ANS: C The articulation of the mandible and the temporal bone is the temporomandibular joint. The other responses are not correct.

A 30-year-old woman tells the nurse that she has been very unsteady and has had difficulty in maintaining her balance. Which area of the brain that is related to these findings would concern the nurse? a. Thalamus b. Brainstem c. Cerebellum d. Extrapyramidal tract

ANS: C The cerebellar system coordinates movement, maintains equilibrium, and helps maintain posture. The thalamus is the primary relay station where sensory pathways of the spinal cord, cerebellum, and brainstem form synapses on their way to the cerebral cortex. The brainstem consists of the midbrain, pons, and medulla and has various functions, especially concerning autonomic centers. The extrapyramidal tract maintains muscle tone for gross automatic movements, such as walking.

The nurse is preparing for an internal genitalia examination of a woman. Which order of the examination is correct? a. Bimanual, speculum, and rectovaginal b. Speculum, rectovaginal, and bimanual c. Speculum, bimanual, and rectovaginal d. Rectovaginal, bimanual, and speculum

ANS: C The correct sequence is speculum examination, then bimanual examination after removing the speculum, and then rectovaginal examination. The examiner should change gloves before performing the rectovaginal examination to avoid spreading any possible infection.

During an internal examination of a woman's genitalia, the nurse will use which technique for proper insertion of the speculum? a. The woman is instructed to bear down, the speculum blades are opened and applied in a swift, upward movement. b. The blades of the speculum are inserted on a horizontal plane, turning them to a 30-degree angle while continuing to insert them. The woman is asked to bear down after the speculum is inserted. c. The woman is instructed to bear down, the width of the blades are horizontally turned, and the speculum is inserted downward at a 45-degree angle toward the small of the womans back. d. The blades are locked open by turning the thumbscrew. Once the blades are open, pressure is applied to the introitus and the blades are inserted downward at a 45-degree angle to bring the cervix into view.

ANS: C The examiner should instruct the woman to bear down, turn the width of the blades horizontally, and insert the speculum at a 45-degree angle downward toward the small of the womans back.

A patient is visiting the clinic for an evaluation of a swollen, painful knuckle. The nurse notices that the knuckle above his ring on the left hand is swollen and that he is unable to remove his wedding ring. This joint is called the _________ joint. a. Interphalangeal b. Tarsometatarsal c. Metacarpophalangeal d. Tibiotalar

ANS: C The joint located just above the ring on the finger is the metacarpophalangeal joint. The interphalangeal joint is located distal to the metacarpophalangeal joint. The tarsometatarsal and tibiotalar joints are found in the foot and ankle.

A patient who is in her first trimester of pregnancy tells the nurse that she is experiencing significant nausea and vomiting and asks when it will improve. The nurse should reply: a. Did your mother have significant nausea and vomiting? b. Many women experience nausea and vomiting until the third trimester. c. Usually, by the beginning of the second trimester, the nausea and vomiting improve. d. At approximately the time you begin to feel the baby move, the nausea and vomiting will subside.

ANS: C The nausea, vomiting, and fatigue of pregnancy improve by the 12th week. Quickening, when the mother recognizes fetal movement, occurs at approximately 18 to 20 weeks.

When the nurse is testing the triceps reflex, what is the expected response? a. Flexion of the hand b. Pronation of the hand c. Extension of the forearm d. Flexion of the forearm

ANS: C The normal response of the triceps reflex is extension of the forearm. The normal response of the biceps reflex causes flexion of the forearm. The other responses are incorrect.

Which statement would be most appropriate when the nurse is introducing the topic of sexual relationships during an interview? a. Now, it is time to talk about your sexual history. When did you first have intercourse? b. Women often feel dissatisfied with their sexual relationships. Would it be okay to discuss this now? c. Women often have questions about their sexual relationship and how it affects their health. Do you have any questions? d. Most women your age have had more than one sexual partner. How many would you say you have had?

ANS: C The nurse should begin with an open-ended question to assess individual needs. The nurse should include appropriate questions as a routine part of the health history, because doing so communicates that the nurse accepts the individuals sexual activity and believes it is important. The nurses comfort with the discussion prompts the patients interest and, possibly, relief that the topic has been introduced. The initial discussion establishes a database for comparison with any future sexual activities and provides an opportunity to screen sexual problems.

The nurse is examining only the rectal area of a woman and should place the woman in what position? a. Lithotomy b. Prone c. Left lateral decubitus d. Bending over the table while standing

ANS: C The nurse should place the female patient in the lithotomy position if the genitalia are being examined as well. The left lateral decubitus position is used for the rectal area alone.

When assessing the neonate, the nurse should test for hip stability with which method? a. Eliciting the Moro reflex b. Performing the Romberg test c. Checking for the Ortolani sign d. Assessing the stepping reflex

ANS: C The nurse should test for hip stability in the neonate by testing for the Ortolani sign. The other tests are not appropriate for testing hip stability.

While assessing a patient who is hospitalized and bedridden, the nurse notices that the patient has been incontinent of stool. The stool is loose and gray-tan in color. The nurse recognizes that this finding indicates which of the following? a. Occult blood b. Inflammation c. Absent bile pigment d. Ingestion of iron preparations

ANS: C The presence of gray-tan stool indicates absent bile pigment, which can occur with obstructive jaundice. The ingestion of iron preparations and the presence of occult blood turns the stools to a black color. Jellylike mucus shreds mixed in the stool would indicate inflammation.

A patient with a lack of oxygen to his heart will have pain in his chest and possibly in the shoulder, arms, or jaw. The nurse knows that the best explanation why this occurs is which one of these statements? a. A problem exists with the sensory cortex and its ability to discriminate the location. b. The lack of oxygen in his heart has resulted in decreased amount of oxygen to the areas experiencing the pain. c. The sensory cortex does not have the ability to localize pain in the heart; consequently, the pain is felt elsewhere. d. A lesion has developed in the dorsal root, which is preventing the sensation from being transmitted normally.

ANS: C The sensory cortex is arranged in a specific pattern, forming a corresponding map of the body. Pain in the right hand is perceived at a specific spot on the map. Some organs, such as the heart, liver, and spleen, are absent from the brain map. Pain originating in these organs is referred because no felt image exists in which to have pain. Pain is felt by proxy, that is, by another body part that does have a felt image. The other responses are not correct explanations.

A 46-year-old man requires an assessment of his sigmoid colon. Which instrument or technique is most appropriate for this examination? a. Proctoscope b. Ultrasound c. Colonoscope d. Rectal examination with an examining finger

ANS: C The sigmoid colon is 40 cm long, and the nurse knows that it is accessible to examination only with the colonoscope. The other responses are not appropriate for an examination of the entire sigmoid colon.

The area of the nervous system that is responsible for mediating reflexes is the: a. Medulla. b. Cerebellum. c. Spinal cord. d. Cerebral cortex.

ANS: C The spinal cord is the main highway for ascending and descending fiber tracts that connect the brain to the spinal nerves; it is responsible for mediating reflexes.

While gathering equipment after an injection, a nurse accidentally received a prick from an improperly capped needle. To interpret this sensation, which of these areas must be intact? a. Corticospinal tract, medulla, and basal ganglia b. Pyramidal tract, hypothalamus, and sensory cortex c. Lateral spinothalamic tract, thalamus, and sensory cortex d. Anterior spinothalamic tract, basal ganglia, and sensory cortex

ANS: C The spinothalamic tract contains sensory fibers that transmit the sensations of pain, temperature, and crude or light touch. Fibers carrying pain and temperature sensations ascend the lateral spinothalamic tract, whereas the sensations of crude touch form the anterior spinothalamic tract. At the thalamus, the fibers synapse with another sensory neuron, which carries the message to the sensory cortex for full interpretation. The other options are not correct.

An 18-year-old patient is having her first pelvic examination. Which action by the nurse is appropriate? a. Inviting her mother to be present during the examination b. Avoiding the lithotomy position for this first time because it can be uncomfortable and embarrassing c. Raising the head of the examination table and giving her a mirror so that she can view the examination d. Fully draping her, leaving the drape between her legs elevated to avoid embarrassing her with eye contact

ANS: C The techniques of the educational or mirror pelvic examination should be used. This is a routine examination with some modifications in attitude, position, and communication. First, the woman is considered an active participant, one who is interested in learning and in sharing decisions about her own health care. The woman props herself up on one elbow, or the head of the table is raised. Her other hand holds a mirror between her legs, above the examiners hands. The young woman can see all that the examiner is doing and has a full view of her genitalia. The mirror works well for teaching normal anatomy and its relationship to sexual behavior. The examiner can ask her if she would like to have a family member, friend, or chaperone present for the examination. The drape should be pushed down between the patients legs so that the nurse can see her face.

To palpate the temporomandibular joint, the nurses fingers should be placed in the depression __________ of the ear. a. Distal to the helix b. Proximal to the helix c. Anterior to the tragus d. Posterior to the tragus

ANS: C The temporomandibular joint can be felt in the depression anterior to the tragus of the ear. The other locations are not correct.

During an examination, the nurse notices that a male patient has a red, round, superficial ulcer with a yellowish serous discharge on his penis. On palpation, the nurse finds a nontender base that feels like a small button between the thumb and fingers. At this point the nurse suspects that this patient has: a. Genital warts. b. Herpes infection. c. Syphilitic chancre. d. Carcinoma lesion.

ANS: C This lesion indicates syphilitic chancre, which begins within 2 to 4 weeks of infection.

During an assessment of the newborn, the nurse expects to see which finding when the anal area is slightly stroked? a. Jerking of the legs b. Flexion of the knees c. Quick contraction of the sphincter d. Relaxation of the external sphincter

ANS: C To assess sphincter tone, the nurse should check the anal reflex by gently stroking the anal area and noticing a quick contraction of the sphincter. The other responses are not correct.

A patients pregnancy test is positive, and she wants to know when the baby is due. The first day of her last menstrual period was June 14, and that period ended June 20. Using the Ngele rule, what is her expected date of delivery? a. March 7 b. March 14 c. March 21 d. March 27

ANS: C To determine the expected date of delivery using the Ngele rule, 7 days are added to the first day of the last menstrual period; then 3 months are subtracted. Therefore, adding 7 days to June 14 would be June 21 and subtracting 3 months would make the expected delivery date March 21. (1st day of LMP + 7 days) - 3 months

The nurse notices that a patient has ulcerations on the tips of the toes and on the lateral aspect of the ankles. This finding indicates: a. Lymphedema. b. Raynaud disease. c. Arterial insufficiency. d. Venous insufficiency.

ANS: C Ulcerations on the tips of the toes and lateral aspect of the ankles are indicative of arterial insufficiency

The nurse is performing an assessment on a 29-year-old woman who visits the clinic complaining of always dropping things and falling down. While testing rapid alternating movements, the nurse notices that the woman is unable to pat both of her knees. Her response is extremely slow and she frequently misses. What should the nurse suspect? a. Vestibular disease b. Lesion of CN IX c. Dysfunction of the cerebellum d. Inability to understand directions

ANS: C When a person tries to perform rapid, alternating movements, responses that are slow, clumsy, and sloppy are indicative of cerebellar disease. The other responses are incorrect.

At the beginning of rounds when entering the room, what should the nurse do first? a. Check the intravenous (IV) infusion site for swelling or redness. b. Check the infusion pump settings for accuracy. c. Make eye contact with the patient, and introduce him or herself as the patients nurse. d. Offer the patient something to drink.

ANS: C When entering a patients room, the nurse should make direct eye contact, without being distracted by IV pumps and other equipment, and introduce him or herself as the patients nurse.

A 46-year-old woman is in the clinic for her annual gynecologic examination. She voices a concern about ovarian cancer because her mother and sister died of it. Which statement does the nurse know to be correct regarding ovarian cancer? a. Ovarian cancer rarely has any symptoms. b. The Pap smear detects the presence of ovarian cancer. c. Women at high risk for ovarian cancer should have annual transvaginal ultrasonography for screening. d. Women over age 40 years should have a thorough pelvic examination every 3 years.

ANS: C With ovarian cancer, the patient may have abdominal pain, pelvic pain, increased abdominal size, bloating, and nonspecific gastrointestinal symptoms; or she may be asymptomatic. The Pap smear does not detect the presence of ovarian cancer. Annual transvaginal ultrasonography may detect ovarian cancer at an earlier stage in women who are at high risk for developing it.

The nurse is testing superficial reflexes on an adult patient. When stroking up the lateral side of the sole and across the ball of the foot, the nurse notices the plantar flexion of the toes. How should the nurse document this finding? a. Positive Babinski sign b. Plantar reflex abnormal c. Plantar reflex present d. Plantar reflex 2+ on a scale from 0 to 4+

ANS: C With the same instrument, the nurse should draw a light stroke up the lateral side of the sole of the foot and across the ball of the foot, similar to an upside-down J. The normal response is plantar flexion of the toes and sometimes of the entire foot. A positive Babinski sign is abnormal and occurs with the response of dorsiflexion of the big toe and fanning of all toes. The plantar reflex is not graded on a 0 to 4+ scale.

During a group prenatal teaching session, the nurse teaches Kegel exercises. Which statements would be appropriate for this teaching session? Select all that apply. a. Kegel exercises help keep your uterus strong during the pregnancy. b. Kegel exercises should be performed twice a day. c. Kegel exercises should be performed 50 to 100 times a day. d. To perform Kegel exercises, slowly squeeze to a peak at the count of eight, and then slowly release to a count of eight. e. To perform Kegel exercises, rapidly perform alternating squeeze-release exercises up to the count of eight.

ANS: C, D Kegel exercises can be performed to prepare for and to recover from birth. The nurse should direct the woman to squeeze slowly to a peak at the count of eight and then to release slowly to the count of eight. The nurse can prescribe this exercise to be performed 50 to 100 times a day.

While the nurse is taking the history of a 68-year-old patient who sustained a head injury 3 days earlier, he tells the nurse that he is on a cruise ship and is 30 years old. The nurse knows that this finding is indicative of a(n): a. Great sense of humor. b. Uncooperative behavior. c. Inability to understand questions. d. Decreased level of consciousness.

ANS: D A change in consciousness may be subtle. The nurse should notice any decreasing level of consciousness, disorientation, memory loss, uncooperative behavior, or even complacency in a previously combative person. The other responses are incorrect.

The nurse knows that a common assessment finding in a boy younger than 2 years old is: a. Inflamed and tender spermatic cord. b. Presence of a hernia in the scrotum. c. Penis that looks large in relation to the scrotum. d. Presence of a hydrocele, or fluid in the scrotum.

ANS: D A common scrotal finding in boys younger than 2 years of age is a hydrocele, or fluid in the scrotum. The other options are not correct.

A young swimmer comes to the sports clinic complaining of a very sore shoulder. He was running at the pool, slipped on some wet concrete, and tried to catch himself with his outstretched hand. He landed on his outstretched hand and has not been able to move his shoulder since. The nurse suspects: a. Joint effusion. b. Tear of rotator cuff. c. Adhesive capsulitis. d. Dislocated shoulder.

ANS: D A dislocated shoulder occurs with trauma involving abduction, extension, and external rotation (e.g., falling on an outstretched arm or diving into a pool).

While performing a rectal examination, the nurse notices a firm, irregularly shaped mass. What should the nurse do next? a. Continue with the examination, and document the finding in the chart. b. Instruct the patient to return for a repeat assessment in 1 month. c. Tell the patient that a mass was felt, but it is nothing to worry about. d. Report the finding, and refer the patient to a specialist for further examination.

ANS: D A firm or hard mass with an irregular shape or rolled edges may signify carcinoma. Any mass that is discovered should be promptly reported for further examination. The other responses are not correct.

A 62-year-old man states that his physician told him that he has an inguinal hernia. He asks the nurse to explain what a hernia is. The nurse should: a. Tell him not to worry and that most men his age develop hernias. b. Explain that a hernia is often the result of prenatal growth abnormalities. c. Refer him to his physician for additional consultation because the physician made the initial diagnosis. d. Explain that a hernia is a loop of bowel protruding through a weak spot in the abdominal muscles.

ANS: D A hernia is a loop of bowel protruding through a weak spot in the musculature. The other options are not correct responses to the patients question.

Which of these statements about the peripheral nervous system is correct? a. The CNs enter the brain through the spinal cord. b. Efferent fibers carry sensory input to the central nervous system through the spinal cord. c. The peripheral nerves are inside the central nervous system and carry impulses through their motor fibers. d. The peripheral nerves carry input to the central nervous system by afferent fibers and away from the central nervous system by efferent fibers.

ANS: D A nerve is a bundle of fibers outside of the central nervous system. The peripheral nerves carry input to the central nervous system by their sensory afferent fibers and deliver output from the central nervous system by their efferent fibers. The other responses are not related to the peripheral nervous system.

During the examination of a patients mouth, the nurse observes a nodular bony ridge down the middle of the hard palate. The nurse would chart this finding as: a. Cheilosis. b. Leukoplakia. c. Ankyloglossia. d. Torus palatinus.

ANS: D A normal variation of the hard palate is a nodular bony ridge down the middle of the hard palate; this variation is termed torus palatinus

During an assessment of a 20-year-old man, the nurse finds a small palpable lesion with a tuft of hair located directly over the coccyx. The nurse knows that this lesion would most likely be a: a. Rectal polyp. b. Pruritus ani. c. Carcinoma. d. Pilonidal cyst.

ANS: D A pilonidal cyst or sinus is a hair-containing cyst or sinus located in the midline over the coccyx or lower sacrum. It often opens as a dimple with a visible tuft of hair and, possibly, an erythematous halo.

The nurse has completed the musculoskeletal examination of a patients knee and has found a positive bulge sign. The nurse interprets this finding to indicate: a. Irregular bony margins. b. Soft-tissue swelling in the joint. c. Swelling from fluid in the epicondyle. d. Swelling from fluid in the suprapatellar pouch.

ANS: D A positive bulge sign confirms the presence of swelling caused by fluid in the suprapatellar pouch. The other options are not correct.

The nurse is providing patient education for a man who has been diagnosed with a rotator cuff injury. The nurse knows that a rotator cuff injury involves the: a. Nucleus pulposus. b. Articular processes. c. Medial epicondyle. d. Glenohumeral joint.

ANS: D A rotator cuff injury involves the glenohumeral joint, which is enclosed by a group of four powerful muscles and tendons that support and stabilize it. The nucleus pulposus is located in the center of each intervertebral disk. The articular processes are projections in each vertebral disk that lock onto the next vertebra, thereby stabilizing the spinal column. The medial epicondyle is located at the elbow.

The nurse is documenting the assessment of an infant. During the abdominal assessment, the nurse noticed a very loud splash auscultated over the upper abdomen when the nurse rocked her from side to side. This finding would indicate: a. Epigastric hernia. b. Pyloric obstruction. c. Hypoactive bowel sounds. d. Hyperactive bowel sounds.

ANS: D A succussion splash, which is unrelated to peristalsis, is a very loud splash auscultated over the upper abdomen when the infant is rocked side to side. It indicates increased air and fluid in the stomach as observed with pyloric obstruction or large hiatus hernia

A 15-year-old boy is seen in the clinic for complaints of dull pain and pulling in the scrotal area. On examination, the nurse palpates a soft, irregular mass posterior to and above the testis on the left. This mass collapses when the patient is supine and refills when he is upright. This description is consistent with: a. Epididymitis. b. Spermatocele. c. Testicular torsion. d. Varicocele.

ANS: D A varicocele consists of dilated, tortuous varicose veins in the spermatic cord caused by incompetent valves within the vein. Symptoms include dull pain or a constant pulling or dragging feeling, or the individual may be asymptomatic. When palpating the mass, the examiner will feel a soft, irregular mass posterior to and above the testis that collapses when the individual is supine and refills when the individual is upright.

When the nurse asks a 68-year-old patient to stand with his feet together and arms at his side with his eyes closed, he starts to sway and moves his feet farther apart. The nurse would document this finding as: a. Ataxia. b. Lack of coordination. c. Negative Homan's sign. d. Positive Romberg sign.

ANS: D Abnormal findings for the Romberg test include swaying, falling, and a widening base of the feet to avoid falling. A positive Romberg sign is a loss of balance that is increased by the closing of the eyes. Ataxia is an uncoordinated or unsteady gait. Homans sign is used to test the legs for deep-vein thrombosis.

A 70-year-old woman tells the nurse that every time she gets up in the morning or after she's been sitting, she gets really dizzy and feels like she is going to fall over. The nurses best response would be: a. Have you been extremely tired lately? b. You probably just need to drink more liquids. c. I'll refer you for a complete neurologic examination. d. You need to get up slowly when you've been lying down or sitting.

ANS: D Aging is accompanied by a progressive decrease in cerebral blood flow. In some people, this decrease causes dizziness and a loss of balance with a position change. These individuals need to be taught to get up slowly. The other responses are incorrect.

Which of these statements is true regarding the penis? a. The urethral meatus is located on the ventral side of the penis. b. The prepuce is the fold of foreskin covering the shaft of the penis. c. The penis is made up of two cylindrical columns of erectile tissue. d. The corpus spongiosum expands into a cone of erectile tissue called the glans.

ANS: D At the distal end of the shaft, the corpus spongiosum expands into a cone of erectile tissue, the glans. The penis is made up of three cylindrical columns of erectile tissue. The skin that covers the glans of the penis is the prepuce. The urethral meatus forms at the tip of the glans.

A 35-year-old woman is at the clinic for a gynecologic examination. During the examination, she asks the nurse, How often do I need to have this Pap test done? Which reply by the nurse is correct? a. It depends. Do you smoke? b. A Pap test needs to be performed annually until you are 65 years of age. c. If you have two consecutive normal Pap tests, then you can wait 5 years between tests. d. After age 30 years, if you have three consecutive normal Pap tests, then you may be screened every 2 to 3 years.

ANS: D Cervical cancer screening with the Pap test continues annually until age 30 years. After age 21, regardless of sexual history or activity, women should be screened every 3 years until age 30, then every 5 years until age 65.

A 32-year-old woman tells the nurse that she has noticed very sudden, jerky movements mainly in her hands and arms. She says, They seem to come and go, primarily when I am trying to do something. I haven't noticed them when Im sleeping. This description suggests: a. Tics. b. Athetosis. c. Myoclonus. d. Chorea.

ANS: D Chorea is characterized by sudden, rapid, jerky, purposeless movements that involve the limbs, trunk, or face. Chorea occurs at irregular intervals, and the movements are all accentuated by voluntary actions.

The nurse documents that a patient has coarse, thickened skin and brown discoloration over the lower legs. Pulses are present. This finding is probably the result of: a. Lymphedema. b. Raynaud disease. c. Chronic arterial insufficiency. d. Chronic venous insufficiency.

ANS: D Chronic venous insufficiency would exhibit firm brawny edema, coarse thickened skin, normal pulses, and brown discoloration

The nurse notices that a woman in an exercise class is unable to jump rope. The nurse is aware that to jump rope, ones shoulder has to be capable of: a. Inversion. b. Supination. c. Protraction. d. Circumduction.

ANS: D Circumduction is defined as moving the arm in a circle around the shoulder. The other options are not correct

When assessing muscle strength, the nurse observes that a patient has complete range of motion against gravity with full resistance. What grade of muscle strength should the nurse record using a 0- to 5-point scale? a. 2 b. 3 c. 4 d. 5

ANS: D Complete range of motion against gravity is normal muscle strength and is recorded as grade 5 muscle strength. The other options are not correct.

During an examination, the nurse notices that a patient is unable to stick out his tongue. Which cranial nerve is involved with the successful performance of this action? a. I b. V c. XI d. XII

ANS: D Cranial nerve XII enables the person to stick out his or her tongue.

The nurse is giving report to the next shift and is using the situation, background, assessment, recommendation (SBAR) framework for communication. Which of these statements reflects the Background portion of the report? a. I'm worried that his gastrointestinal bleeding is getting worse. b. We need an order for oxygen. c. My name is Ms. Smith, and Im giving the report on Mrs. X in room 1104. d. He is 4 days postoperative, and his incision is open to air.

ANS: D During the Background portion, the nurse should state data pertinent to the moments problem such as the condition of the patients incision. During the Situation portion, the nurse provides his or her name and the patients name. During the Assessment portion, the nurse states what he or she thinks is happening (e.g., gastrointestinal bleeding). During the Recommendation portion, the nurse should offer probable solutions or orders that may be implemented.

A woman who is 28 weeks pregnant has bilateral edema in her lower legs after working 8 hours a day as a cashier at a local grocery store. She is worried about her legs. What is the nurses best response? a. You will be at risk for development of varicose veins when your legs are edematous. b. I would like to listen to your heart sounds. Edema can indicate a problem with your heart. c. Edema is usually the result of too much salt and fluids in your diet. You may need to cut down on salty foods. d. As your baby grows, it slows blood return from your legs, causing the swelling. This often occurs with prolonged standing.

ANS: D Edema of the lower extremities occurs because of the enlarging fetus, which impairs venous return. Prolonged standing worsens the edema. Typically, the bilateral, dependent edema experienced with pregnancy is not the result of a cardiac pathologic condition.

The nurse has just completed an examination of a patients extraocular muscles. When documenting the findings, the nurse should document the assessment of which cranial nerves? a. II, III, and VI b. II, IV, and V c. III, IV, and V d. III, IV, and VI

ANS: D Extraocular muscles are innervated by cranial nerves III, IV, and VI.

The nurse is palpating the abdomen of a woman who is 35 weeks pregnant and notices that the fetal head is facing downward toward the pelvis. The nurse would document this as fetal: a. Lie. b. Variety. c. Attitude. d. Presentation.

ANS: D Fetal presentation describes the part of the fetus that is entering the pelvis first. Fetal lie is orientation of the fetal spine to the maternal spine. Fetal attitude is the position of fetal parts in relation to each other, and fetal variety is the location of the fetal back to the maternal pelvis.

Fibrous bands running directly from one bone to another that strengthen the joint and help prevent movement in undesirable directions are called: a. Bursa. b. Tendons. c. Cartilage. d. Ligaments.

ANS: D Fibrous bands running directly from one bone to another that strengthen the joint and help prevent movement in undesirable directions are called ligaments. The other options are not correct.

A 22-year-old woman is being seen at the clinic for problems with vulvar pain, dysuria, and fever. On physical examination, the nurse notices clusters of small, shallow vesicles with surrounding erythema on the labia. Inguinal lymphadenopathy present is also present. The most likely cause of these lesions is: a. Pediculosis pubis. b. Contact dermatitis. c. HPV. d. Herpes simplex virus type 2.

ANS: D Herpes simplex virus type 2 exhibits clusters of small, shallow vesicles with surrounding erythema that erupt on the genital areas. Inguinal lymphadenopathy is also present. The woman reports local pain, dysuria, and fever.

During a health history, a patient tells the nurse that he has trouble in starting his urine stream. This problem is known as: a. Urgency. b. Dribbling. c. Frequency. d. Hesitancy.

ANS: D Hesitancy is trouble in starting the urine stream. Urgency is the feeling that one cannot wait to urinate. Dribbling is the last of the urine before or after the main act of urination. Frequency is urinating more often than usual.

After assessing a female patient, the nurse notices flesh-colored, soft, pointed, moist, papules in a cauliflower-like patch around her introitus. This finding is most likely: a. Urethral caruncle. b. Syphilitic chancre. c. Herpes simplex virus. d. Human papillomavirus.

ANS: D Human papillomavirus appears in a flesh-colored, soft, moist, cauliflower-like patch of papules

In assessing a 70-year-old patient who has had a recent cerebrovascular accident, the nurse notices right-sided weakness. What might the nurse expect to find when testing his reflexes on the right side? a. Lack of reflexes b. Normal reflexes c. Diminished reflexes d. Hyperactive reflexes

ANS: D Hyperreflexia is the exaggerated reflex observed when the monosynaptic reflex arc is released from the influence of higher cortical levels. This response occurs with upper motor neuron lesions (e.g., a cerebrovascular accident). The other responses are incorrect

During the assessment of a woman in her 22nd week of pregnancy, the nurse is unable to hear fetal heart tones with the fetoscope. The nurse should: a. Immediately notify the physician, then wait 10 minutes and try again. b. Ask the woman if she has felt the baby move today. c. Wait 10 minutes, and try again. d. Use ultrasound to verify cardiac activity.

ANS: D If no fetal heart tones are heard during auscultation with a fetoscope, then the nurse should verify cardiac activity using ultrasonography. An ultrasound should be immediately done and before notifying the physician or causing the woman distress by asking about fetal movement.

The nurse is performing a genital examination on a male patient and notices urethral drainage. When collecting urethral discharge for microscopic examination and culture, the nurse should: a. Ask the patient to urinate into a sterile cup. b. Ask the patient to obtain a specimen of semen. c. Insert a cotton-tipped applicator into the urethra. d. Compress the glans between the examiners thumb and forefinger, and collect any discharge.

ANS: D If urethral discharge is noticed, then the examiner should collect a smear for microscopic examination and culture by compressing the glans anteroposteriorly between the thumb and forefinger. The other options are not correct actions.

During an examination, the nurse asks the patient to perform the Valsalva maneuver and notices that the patient has a moist, red, doughnut-shaped protrusion from the anus. The nurse knows that this finding is consistent with a: a. Rectal polyp. b. Hemorrhoid. c. Rectal fissure. d. Rectal prolapse.

ANS: D In rectal prolapse, the rectal mucous membrane protrudes through the anus, appearing as a moist red doughnut with radiating lines. It occurs after a Valsalva maneuver, such as straining at passing stool or with exercising

A 65-year-old woman is in the office for routine gynecologic care. She had a complete hysterectomy 3 months ago after cervical cancer was detected. Which statement does the nurse know to be true regarding this visit? a. Her cervical mucosa will be red and dry looking. b. She will not need to have a Pap smear performed. c. The nurse can expect to find that her uterus will be somewhat enlarged and her ovaries small and hard. d. The nurse should plan to lubricate the instruments and the examining hand adequately to avoid a painful examination.

ANS: D In the aging adult woman, natural lubrication is decreased; therefore, to avoid a painful examination, the nurse should take care to lubricate the instruments and the examining hand adequately. Menopause, with the resulting decrease in estrogen production, shows numerous physical changes. The cervix shrinks and looks pale and glistening. With the bimanual examination, the uterus feels smaller and firmer and the ovaries are not normally palpable. Women should continue cervical cancer screening up to age 65 years if they have an intact cervix and are in good health. Women who have had a total hysterectomy do not need cervical cancer screening if they have 3 consecutive negative Pap tests or 2 or more consecutive negative HIV and Pap tests within the last 10 years.

The nurse is explaining to a patient that there are shock absorbers in his back to cushion the spine and to help it move. The nurse is referring to his: a. Vertebral column. b. Nucleus pulposus. c. Vertebral foramen. d. Intervertebral disks.

ANS: D Intervertebral disks are elastic fibrocartilaginous plates that cushion the spine similar to shock absorbers and help it move. The vertebral column is the spinal column itself. The nucleus pulposus is located in the center of each disk. The vertebral foramen is the channel, or opening, for the spinal cord in the vertebrae.

The nurse is examining a 3-month-old infant. While the nurse holds his or her thumbs on the infants inner mid thighs and the fingers on the outside of the infants hips, touching the greater trochanter, the nurse adducts the legs until the his or her thumbs touch and then abducts the legs until the infants knees touch the table. The nurse does not notice any clunking sounds and is confident to record a: a. Positive Allis test. b. Negative Allis test. c. Positive Ortolani sign. d. Negative Ortolani sign.

ANS: D Normally, this maneuver feels smooth and has no sound. With a positive Ortolani sign, however, the nurse will feel and hear a clunk, as the head of the femur pops back into place. A positive Ortolani sign also reflects hip instability. The Allis test also tests for hip dislocation but is performed by comparing leg lengths.

A 22-year-old woman has been considering using oral contraceptives. As a part of her health history, the nurse should ask: a. Do you have a history of heart murmurs? b. Will you be in a monogamous relationship? c. Have you carefully thought this choice through? d. If you smoke, how many cigarettes do you smoke per day?

ANS: D Oral contraceptives, together with cigarette smoking, increase the risk for cardiovascular side effects. If cigarettes are used, then the nurse should assess the patients smoking history. The other questions are not appropriate.

While examining a 48-year-old patients eyes, the nurse notices that he had to move the handheld vision screener farther away from his face. The nurse would suspect: a. Myopia. b. Omniopia. c. Hyperopia. d. Presbyopia.

ANS: D Presbyopia, the decrease in power of accommodation with aging, is suggested when the handheld vision screener card is moved farther away

A female patient has nausea, breast tenderness, fatigue, and amenorrhea. Her last menstrual period was 6 weeks ago. The nurse interprets that this patient is experiencing __________ signs of pregnancy. a. Positive b. Possible c. Probable d. Presumptive

ANS: D Presumptive signs of pregnancy are those that the woman experiences and include amenorrhea, breast tenderness, fatigue, nausea, and increased urinary frequency. Probable signs are those that are detected by the examiner, such as an enlarged uterus or changes in the cervix. Positive signs of pregnancy are those that document direct evidence of the fetus such as fetal heart tones or positive cardiac activity on ultrasound.

The nurse is providing patient teaching about an erectile dysfunction drug. One of the drugs potential side effects is prolonged, painful erection of the penis without sexual stimulation, which is known as: a. Orchitis. b. Stricture. c. Phimosis. d. Priapism.

ANS: D Priapism is prolonged, painful erection of the penis without sexual desire. Orchitis is inflammation of the testes. Stricture is a narrowing of the opening of the urethral meatus. Phimosis is the inability to retract the foreskin.

Which of these statements best describes the action of the hormone progesterone during pregnancy? a. Progesterone produces the hormone human chorionic gonadotropin. b. Duct formation in the breast is stimulated by progesterone. c. Progesterone promotes sloughing of the endometrial wall. d. Progesterone maintains the endometrium around the fetus.

ANS: D Progesterone prevents the sloughing of the endometrial wall and maintains the endometrium around the fetus. Progesterone increases the alveoli in the breast and keeps the uterus in a quiescent state. The other options are not correct.

A woman who is 8 weeks pregnant is in the clinic for a checkup. The nurse reads on her chart that her cervix is softened and looks cyanotic. The nurse knows that the woman is exhibiting __________ sign and __________ sign. a. Tanner; Hegar b. Hegar; Goodell c. Chadwick; Hegar d. Goodell; Chadwick

ANS: D Shortly after the first missed menstrual period, the female genitalia show signs of the growing fetus. The cervix softens (Goodell sign) at 4 to 6 weeks, and the vaginal mucosa and cervix look cyanotic (Chadwick sign) at 8 to 12 weeks. These changes occur because of increased vascularity and edema of the cervix and hypertrophy and hyperplasia of the cervical glands. Hegar sign occurs when the isthmus of the uterus softens at 6 to 8 weeks. Tanner sign is not a correct response.

A woman in her second trimester of pregnancy complains of heartburn and indigestion. When discussing this with the woman, the nurse considers which explanation for these problems? a. Tone and motility of the gastrointestinal tract increase during the second trimester. b. Sluggish emptying of the gallbladder, resulting from the effects of progesterone, often causes heartburn. c. Lower blood pressure at this time decreases blood flow to the stomach and gastrointestinal tract. d. Enlarging uterus and altered esophageal sphincter tone predispose the woman to have heartburn.

ANS: D Stomach displacement from the enlarging uterus plus altered esophageal sphincter and gastric tone as a result of progesterone predispose the woman to heartburn. The tone and motility of the gastrointestinal tract are decreased, not increased, during pregnancy. Emptying of the gallbladder may become more sluggish during pregnancy but is not related to indigestion. Rather, some women are predisposed to gallstone formation. A lower blood pressure may occur during the second semester, but it does not affect digestion.

A 68-year-old woman has come in for an assessment of her rheumatoid arthritis, and the nurse notices raised, firm, nontender nodules at the olecranon bursa and along the ulna. These nodules are most commonly diagnosed as: a. Epicondylitis. b. Gouty arthritis. c. Olecranon bursitis. d. Subcutaneous nodules.

ANS: D Subcutaneous nodules are raised, firm, and nontender and occur with rheumatoid arthritis in the olecranon bursa and along the extensor surface of the ulna.

The nurse is palpating the uterus of a woman who is 8 weeks pregnant. Which finding would be considered to be most consistent with this stage of pregnancy? a. The uterus seems slightly enlarged and softened. b. It reaches the pelvic brim and is approximately the size of a grapefruit. c. The uterus rises above the pelvic brim and is approximately the size of a cantaloupe. d. It is about the size of an avocado, approximately 8 cm across the fundus.

ANS: D The 8-week pregnant uterus is approximately the size of an avocado, 7 to 8 cm across the fundus. The 6-week pregnant uterus is slightly enlarged and softened. The 10-week pregnant uterus is approximately the size of a grapefruit and may reach the pelvic brim. The 12-week pregnant uterus will fill the pelvis. At 12 weeks, the uterus is sized from the abdomen.

During a health history, a 22-year old woman asks, Can I get that vaccine for human papilloma virus (HPV)? I have genital warts and Id like them to go away! What is the nurses best response? a. The HPV vaccine is for girls and women ages 9 to 26 years, so we can start that today. b. This vaccine is only for girls who have not yet started to become sexually active. c. Let's check with the physician to see if you are a candidate for this vaccine. d. The vaccine cannot protect you if you already have an HPV infection.

ANS: D The HPV vaccine is appropriate for girls and women age 9 to 26 years and is administered to prevent cervical cancer by preventing HPV infections before girls become sexually active. However, it cannot protect the woman if an HPV infection is already present.

Which statement concerning the anal canal is true? The anal canal: a. Is approximately 2 cm long in the adult. b. Slants backward toward the sacrum. c. Contains hair and sebaceous glands. d. Is the outlet for the gastrointestinal tract.

ANS: D The anal canal is the outlet for the gastrointestinal tract and is approximately 3.8 cm long in the adult. It is lined with a modified skin that does not contain hair or sebaceous glands, and it slants forward toward the umbilicus.

While performing an assessment of the perianal area of a patient, the nurse notices that the pigmentation of anus is darker than the surrounding skin, the anal opening is closed, and a skin sac that is shiny and blue is noted. The patient mentioned that he has had pain with bowel movements and has occasionally noted some spots of blood. What would this assessment and history most likely indicate? a. Anal fistula b. Pilonidal cyst c. Rectal prolapse d. Thrombosed hemorrhoid

ANS: D The anus normally looks moist and hairless, with coarse folded skin that is more pigmented than the perianal skin, and the anal opening is tightly closed. The shiny blue skin sac indicates a thrombosed hemorrhoid.

Which of these correctly describes the average length of pregnancy? a. 38 weeks b. 9 lunar months c. 280 days from the last day of the last menstrual period d. 280 days from the first day of the last menstrual period

ANS: D The average length of pregnancy is 280 days from the first day of the last menstrual period, which is equal to 40 weeks, 10 lunar months, or roughly 9 calendar months.

When the nurse performs the confrontation test, the nurse has assessed: a. Extraocular eye muscles (EOMs). b. Pupils (pupils equal, round, reactive to light, and accommodation [PERRLA]). c. Near vision. d. Visual fields.

ANS: D The confrontation test assesses visual fields. The other options are not tested with the confrontation test.

A professional tennis player comes into the clinic complaining of a sore elbow. The nurse will assess for tenderness at the: a. Olecranon bursa. b. Annular ligament. c. Base of the radius. d. Medial and lateral epicondyle.

ANS: D The epicondyles, the head of the radius, and the tendons are common sites of inflammation and local tenderness, commonly referred to as tennis elbow. The other locations are not affected.

The nurse is testing the function of CN XI. Which statement best describes the response the nurse should expect if this nerve is intact? The patient: a. Demonstrates the ability to hear normal conversation. b. Sticks out the tongue midline without tremors or deviation. c. Follows an object with his or her eyes without nystagmus or strabismus. d. Moves the head and shoulders against resistance with equal strength.

ANS: D The following normal findings are expected when testing the spinal accessory nerve (CN XI): The patients sternomastoid and trapezius muscles are equal in size; the person can forcibly rotate the head both ways against resistance applied to the side of the chin with equal strength; and the patient can shrug the shoulders against resistance with equal strength on both sides. Checking the patients ability to hear normal conversation checks the function of CN VIII. Having the patient stick out the tongue checks the function of CN XII. Testing the eyes for nystagmus or strabismus is performed to check CNs III, IV, and VI.

The nurse is examining the glans and knows which finding is normal for this area? a. The meatus may have a slight discharge when the glans is compressed. b. Hair is without pest inhabitants. c. The skin is wrinkled and without lesions. d. Smegma may be present under the foreskin of an uncircumcised male.

ANS: D The glans looks smooth and without lesions and does not have hair. The meatus should not have any discharge when the glans is compressed. Some cheesy smegma may have collected under the foreskin of an uncircumcised male.

During the assessment of an 18-month-old infant, the mother expresses concern to the nurse about the infants inability to toilet train. What would be the nurses best response? a. Some children are just more difficult to train, so I wouldn't worry about it yet. b. Have you considered reading any of the books on toilet training? They can be very helpful. c. This could mean that there is a problem in your baby's development. We'll watch her closely for the next few months. d. The nerves that will allow your baby to have control over the passing of stools are not developed until at least 18 to 24 months of age.

ANS: D The infant passes stools by reflex. Voluntary control of the external anal sphincter cannot occur until the nerves supplying the area have become fully myelinated, usually around 1 to 2 years of age. Toilet training usually starts after the age of 2 years.

When the nurse is conducting sexual history from a male adolescent, which statement would be most appropriate to use at the beginning of the interview? a. Do you use condoms? b. You don't masturbate, do you? c. Have you had sex in the last 6 months? d. Often adolescents your age have questions about sexual activity.

ANS: D The interview should begin with a permission statement, which conveys that it is normal and acceptable to think or feel a certain way. Sounding judgmental should be avoided.

When performing a genital assessment on a middle-aged man, the nurse notices multiple soft, moist, painless papules in the shape of cauliflower-like patches scattered across the shaft of the penis. These lesions are characteristic of: a. Carcinoma. b. Syphilitic chancres. c. Genital herpes. d. Genital warts.

ANS: D The lesions of genital warts are soft, pointed, moist, fleshy, painless papules that may be single or multiple in a cauliflower-like patch. They occur on the shaft of the penis, behind the corona, or around the anus, where they may grow into large grapelike clusters.

When reviewing the musculoskeletal system, the nurse recalls that hematopoiesis takes place in the: a. Liver. b. Spleen. c. Kidneys. d. Bone marrow.

ANS: D The musculoskeletal system functions to encase and protect the inner vital organs, to support the body, to produce red blood cells in the bone marrow (hematopoiesis), and to store minerals. The other options are not correct.

During an assessment, the nurse is unable to palpate pulses in the left lower leg. What should the nurse do next? a. Document that the pulses are nonpalpable. b. Reassess the pulses in 1 hour. c. Ask the patient turn to the side, and then palpate for the pulses again. d. Use a Doppler device to assess the pulses.

ANS: D The nurse should be prepared to assess pulses in the lower extremities by Doppler measurement if they cannot be detected by palpation.

During the taking of the health history of a 78-year-old man, his wife states that he occasionally has problems with short-term memory loss and confusion: He cant even remember how to button his shirt. When assessing his sensory system, which action by the nurse is most appropriate? a. The nurse would not test the sensory system as part of the examination because the results would not be valid. b. The nurse would perform the tests, knowing that mental status does not affect sensory ability. c. The nurse would proceed with an explanation of each test, making certain that the wife understands. d. Before testing, the nurse would assess the patients mental status and ability to follow directions.

ANS: D The nurse should ensure the validity of the sensory system testing by making certain that the patient is alert, cooperative, comfortable, and has an adequate attention span. Otherwise, the nurse may obtain misleading and invalid results.

A patient states, Whenever I open my mouth real wide, I feel this popping sensation in front of my ears. To further examine this, the nurse would: a. Place the stethoscope over the temporomandibular joint, and listen for bruits. b. Place the hands over his ears, and ask him to open his mouth really wide. c. Place one hand on his forehead and the other on his jaw, and ask him to try to open his mouth. d. Place a finger on his temporomandibular joint, and ask him to open and close his mouth.

ANS: D The nurse should palpate the temporomandibular joint by placing his or her fingers over the joint as the person opens and closes the mouth.

A 50-year-old woman is in the clinic for weakness in her left arm and leg that she has noticed for the past week. The nurse should perform which type of neurologic examination? a. Glasgow Coma Scale b. Neurologic recheck examination c. Screening neurologic examination d. Complete neurologic examination

ANS: D The nurse should perform a complete neurologic examination on an individual who has neurologic concerns (e.g., headache, weakness, loss of coordination) or who is showing signs of neurologic dysfunction. The Glasgow Coma Scale is used to define a persons level of consciousness. The neurologic recheck examination is appropriate for those who are demonstrating neurologic deficits. The screening neurologic examination is performed on seemingly well individuals who have no significant subjective findings from the health history.

During an examination, a patient has just successfully completed the finger-to-nose and the rapid-alternating-movements tests and is able to run each heel down the opposite shin. The nurse will conclude that the patients __________ function is intact. a. Occipital b. Cerebral c. Temporal d. Cerebellar

ANS: D The nurse should test cerebellar function of the upper extremities by using the finger-to-nose test or rapid-alternating-movements test. The nurse should test cerebellar function of the lower extremities by asking the person to run each heel down the opposite shin.

A 13-year-old girl is visiting the clinic for a sports physical examination. The nurse should remember to include which of these tests in the examination? a. Testing for occult blood b. Valsalva maneuver c. Internal palpation of the anus d. Inspection of the perianal area

ANS: D The perianal region of the school-aged child and adolescent should be inspected during the examination of the genitalia. Internal palpation is not routinely performed at this age. Testing for occult blood and performing the Valsalva maneuver are also not necessary.

A 60-year-old man has just been told that he has benign prostatic hypertrophy (BPH). He has a friend who just died from cancer of the prostate. He is concerned this will happen to him. How should the nurse respond? a. The swelling in your prostate is only temporary and will go away. b. We will treat you with chemotherapy so we can control the cancer. c. It would be very unusual for a man your age to have cancer of the prostate. d. The enlargement of your prostate is caused by hormonal changes, and not cancer.

ANS: D The prostate gland commonly starts to enlarge during the middle adult years. BPH is present in 1 in 10 men at the age of 40 years and increases with age. It is believed that the hypertrophy is caused by hormonal imbalance that leads to the proliferation of benign adenomas. The other responses are not appropriate.

During an assessment of a 32-year-old patient with a recent head injury, the nurse notices that the patient responds to pain by extending, adducting, and internally rotating his arms. His palms pronate, and his lower extremities extend with plantar flexion. Which statement concerning these findings is most accurate? This patients response: a. Indicates a lesion of the cerebral cortex. b. Indicates a completely nonfunctional brainstem. c. Is normal and will go away in 24 to 48 hours. d. Is a very ominous sign and may indicate brainstem injury.

ANS: D These findings are all indicative of decerebrate rigidity, which is a very ominous condition and may indicate a brainstem injury.

A 25-year-old woman comes to the emergency department with a sudden fever of 38.3 C and abdominal pain. Upon examination, the nurse notices that she has rigid, boardlike lower abdominal musculature. When the nurse tries to perform a vaginal examination, the patient has severe pain when the uterus and cervix are moved. The nurse knows that these signs and symptoms are suggestive of: a. Endometriosis. b. Uterine fibroids. c. Ectopic pregnancy. d. Pelvic inflammatory disease.

ANS: D These signs and symptoms are suggestive of acute pelvic inflammatory disease, also known as acute salpingitis

During the taking of a health history, the patient states, It really hurts back there, and sometimes it itches, too. I have even seen blood on the tissue when I have a bowel movement. Is there something there? The nurse should expect to see which of these upon examination of the anus? a. Rectal prolapse b. Internal hemorrhoid c. External hemorrhoid that has resolved d. External hemorrhoid that is thrombosed

ANS: D These symptoms are consistent with an external hemorrhoid. An external hemorrhoid, when thrombosed, contains clotted blood and becomes a painful, swollen, shiny blue mass that itches and bleeds with defecation. When the external hemorrhoid resolves, it leaves a flabby, painless skin sac around the anal orifice. An internal hemorrhoid is not palpable but may appear as a red mucosal mass when the person performs a Valsalva maneuver. A rectal prolapse appears as a moist, red doughnut with radiating lines.

A man who has had gout for several years comes to the clinic with a problem with his toe. On examination, the nurse notices the presence of hard, painless nodules over the great toe; one has burst open with a chalky discharge. This finding is known as: a. Callus. b. Plantar wart. c. Bunion. d. Tophi.

ANS: D Tophi are collections of monosodium urate crystals resulting from chronic gout in and around the joint that cause extreme swelling and joint deformity. They appear as hard, painless nodules (tophi) over the metatarsophalangeal joint of the first toe and they sometimes burst with a chalky discharge

The nurse is examining a 2-month-old infant and notices asymmetry of the infants gluteal folds. The nurse should assess for other signs of what disorder? a. Fractured clavicle b. Down syndrome c. Spina bifida d. Hip dislocation

ANS: D Unequal gluteal folds may accompany hip dislocation after 2 to 3 months of age, but some asymmetry may occur in healthy children. Further assessment is needed. The other responses are not correct.

During a woman's 34th week of pregnancy, she is told that she has preeclampsia. The nurse knows which statement concerning preeclampsia is true? a. Preeclampsia has little effect on the fetus. b. Edema is one of the main indications of preeclampsia. c. Eclampsia only occurs before delivery of the baby. d. Untreated preeclampsia may contribute to restriction of fetal growth.

ANS: D Untreated preeclampsia may progress to eclampsia, which is manifested by generalized tonic-clonic seizures. Eclampsia may develop as late as 10 days postpartum. Before the syndrome becomes clinically manifested, it is affecting the placenta through vasospasm and a series of small infarctions. The placentas capacity to deliver oxygen and nutrients may be seriously diminished, and fetal growth may be restricted. Edema is common in pregnancy and is not an indicator of preeclampsia.

Which statement concerning the testes is true? a. The lymphatic vessels of the testes drain into the abdominal lymph nodes. b. The vas deferens is located along the inferior portion of each testis. c. The right testis is lower than the left because the right spermatic cord is longer. d. The cremaster muscle contracts in response to cold and draws the testicles closer to the body.

ANS: D When it is cold, the cremaster muscle contracts, which raises the scrotal sac and brings the testes closer to the body to absorb heat necessary for sperm viability. The lymphatic vessels of the testes drain into the inguinal lymph nodes. The vas deferens is located along the upper portion of each testis. The left testis is lower than the right because the left spermatic cord is longer.

When the nurse is performing a genital examination on a male patient, which action is correct? a. Auscultating for the presence of a bruit over the scrotum b. Palpating for the vertical chain of lymph nodes along the groin, inferior to the inguinal ligament c. Palpating the inguinal canal only if a bulge is present in the inguinal region during inspection d. Having the patient shift his weight onto the left (unexamined) leg when palpating for a hernia on the right side

ANS: D When palpating for the presence of a hernia on the right side, the male patient is asked to shift his weight onto the left (unexamined) leg. Auscultating for a bruit over the scrotum is not appropriate. When palpating for lymph nodes, the horizontal chain is palpated. The inguinal canal should be palpated whether a bulge is present or not.

When the nurse is performing a genital examination on a male patient, the patient has an erection. The nurses most appropriate action or response is to: a. Ask the patient if he would like someone else to examine him. b. Continue with the examination as though nothing has happened. c. Stop the examination, leave the room while stating that the examination will resume at a later time. d. Reassure the patient that this is a normal response and continue with the examination.

ANS: D When the male patient has an erection, the nurse should reassure the patient that this is a normal physiologic response to touch and proceed with the rest of the examination. The other responses are not correct and may be perceived as judgmental.

When performing a genitourinary assessment on a 16-year-old male adolescent, the nurse notices a swelling in the scrotum that increases with increased intra-abdominal pressure and decreases when he is lying down. The patient complains of pain when straining. The nurse knows that this description is most consistent with a(n) ______ hernia. a. Femoral b. Incisional c. Direct inguinal d. Indirect inguinal

ANS: D With indirect inguinal hernias, pain occurs with straining and a soft swelling increases with increased intra-abdominal pressure, which may decrease when the patient lies down. These findings do not describe the other hernias.

A 78-year-old man has a history of a cerebrovascular accident. The nurse notes that when he walks, his left arm is immobile against the body with flexion of the shoulder, elbow, wrist, and fingers and adduction of the shoulder. His left leg is stiff and extended and circumducts with each step. What type of gait disturbance is this individual experiencing? a. Scissors gait b. Cerebellar ataxia c. Parkinsonian gait d. Spastic hemiparesis

ANS: D With spastic hemiparesis, the arm is immobile against the body. Flexion of the shoulder, elbow, wrist, and fingers occurs, and adduction of the shoulder, which does not swing freely, is observed. The leg is stiff and extended and circumducts with each step. Causes of this type of gait include cerebrovascular accident.


Related study sets

3XChapter 15-17 Derivatives and Currency Management

View Set

5 - Life Insurance Underwriting and Policy Issue

View Set

Geology 103 Chapters 1-7 Midterm

View Set

Anatomy & Physiology: Respiratory & Lymphatic Lab Practical

View Set